SlideShare a Scribd company logo
1 of 34
22-year-old Suma, with a history of epilepsy is undergoing the pre-anaesthetic review. The
use of which inhalational agent is contraindicated in this patient?
A: Halothane
B:
Enfluran
e
C: Sevoflurane
D: Isoflurane
Correct Ans:B
Explanation
Enflurane can precipitate generalized tonic clonic seizure in epileptics. It is however safer than
halothane, causes less myocardiac depression and less hypotension.
Sample Previous Year Question on Epilepsy based on previous Year Questions of
NEET PG, USMLE,PLAB,FMGE (MCI Screening). Please visit
www.medicoapps.org for more such Quizzes
A patient presented with short lasting episodic behavioural changes which include agitation
and dream like state with thrashing movements of his limbs. He does not recall these
episodes and has no apparent precipitating factor. Which of the following is the most likely
diagnosis?
A: Panic episodes
B:
Schizophreni
a
C: Temporal lobe epilepsy
D: Dissociative disorder
Correct Ans:C
Explanation
The patient in the question has most likely suffered an attack of temporal lobe epilepsy. The
loss of consciousness need not always be present but it is associated with loss of memory
for the episode. In temporal lobe seizure, the patient usually experiences auras or warning
signs, epigastric discomfort, olfactory hallucination, sensation of deja vu. A dream like state
is often a feature of TLE. There may be loss of consciousness abnormal movement of mouth,
and rarely abnormal movement of body. Frontal lobe epilepsy is also associated with
episodes of agitation known as ‘intermittent explosive disorder’.
Ref: Clinical Neuroanatomy By Stephen G. Waxman, 26th Edition, Chapter 19
Sample Previous Year Question on Epilepsy based on previous Year Questions of
NEET PG, USMLE,PLAB,FMGE (MCI Screening). Please visit
www.medicoapps.org for more such Quizzes
Gustatory hallucinations are most commonly associated with:
A: Temporal lobe epilepsy
B:
Grand mal
epilepsy
C: Anxiety disorders
D: Tobacco dependence
Correct Ans:A
Explanation
Gustatory hallucinations are most commonly associated with temporal lobe lesions,
especially uncinate gyrus seizures. Patients report experiencing bitter, sweet, salty,
tobacco-like, metallic or indescribable strange tastes. They are found in 4% of seizure
patients with temporal lobe foci.
Ref: Clinical neuropsychology 4th Ed By Kenneth M.Heilman, Page 488.
Sample Previous Year Question on Epilepsy based on previous Year Questions of
NEET PG, USMLE,PLAB,FMGE (MCI Screening). Please visit
www.medicoapps.org for more such Quizzes
A 27 year old man with epilepsy presents with complaints of persistent lethargy and
occasional feelings of intoxication, although he does not consume alcohol. The patient is
currently taking phenobarbital for the long-term management of tonic-clonic seizures. In
addition, he was recently started on a medication for the treatment of gastroesophageal
reflux disease. Which of the following agents was he most likely prescribed?
A:
Cimetidin
e
B: Famotidine
C: Lansoprazole
D: Ranitidine
Correct Ans:A
Explanation
Cimetidine is an H2-receptor antagonist indicated for the short-term and maintenance
treatment of duodenal and gastric ulceration, as well as gastroesophageal reflux disease.
One of the primary disadvantages of using this agent, with respect to other H2-receptor
antagonists famotidine and ranitidine, is that it is a relatively potent hepatic enzyme
inhibitor. Therefore, this medication is likely to decrease the metabolism of other
hepatically metabolized medications, such as phenobarbital. When the metabolism of a
medication is decreased, the blood levels will increase, leading to an extension of the
therapeutic effect and/or toxicity. Phenobarbital is a barbiturate indicated for the
treatment of tonic-clonic seizures and status epilepticus. When the blood concentration of
this medication increases, lethargy and feelings of intoxication may occur.
Lansoprazole is a gastric acid proton-pump inhibitor indicated for the short-term and
maintenance treatment of duodenal and gastric ulceration, as well as gastroesophageal
reflux disease. This agent does not affect hepatic enzymes.
Ref: McQuaid K.R. (2012). Chapter 62. Drugs Used in the Treatment of Gastrointestinal
Diseases. In B.G. Katzung, S.B. Masters, A.J. Trevor (Eds),Basic & Clinical Pharmacology,
12e.
Sample Previous Year Question on Epilepsy based on previous Year Questions of
NEET PG, USMLE,PLAB,FMGE (MCI Screening). Please visit
www.medicoapps.org for more such Quizzes
At autopsy, a body is found to have copious fine leathery froth in mouth and
nostrils which increased on pressure over chest. Which of the following was the
most likely cause of death?
A: Epilepsy
B:
Hangin
g
C: Drowning
D: Opium poisoning
Correct Ans:C
Explanation
Presence of fine, white, leathery froth seen at the mouth and nostrils is one of the
most characteristic external signs of drowning. It is white or rarely blood stained,
lather-like, abundant and increase in amount with compression of chest. Even
after wiping it gradually reappears, especially if pressure is applied to the chest.
Froth is also seen in strangulation, acute pulmonary edema, electric shock,
during epileptic fit, in opium poisoning and putrefaction. But in all these cases
the quantity of froth is not as large as in drowning, and the bubbles are also
much smaller.
Ref: Parikh’s Textbook of Medical Jurisprudence, Forensic Medicine and
Toxicology, 6th Edition, Pages 3.66-3.67 ; Essentials of Forensic Medicine and
Toxicology By Dr K S Narayan Reddy, 27th Edition, Pages 325-6
Sample Previous Year Question on Epilepsy based on previous Year Questions of
NEET PG, USMLE,PLAB,FMGE (MCI Screening). Please visit
www.medicoapps.org for more such Quizzes
Corpus callosotomy is useful in treatment of:
A: Epilepsy
B:
Strok
e
C: Alzheimer’s disease
D: none of the above
Correct Ans:A
Explanation
Patients with generalized seizures, atonic seizures associated with drop attacks, or absence
seizures, who are found to have bilaterally coordinated pathologic cortical discharges on
EEG and who fail AED therapy, may be candidates for corpus callosotomy. The corpus
callosum is a large white matter tract that connects the cerebral hemispheres.
Loss of consciousness requires simultaneous seizure activity in both hemispheres. Focal or
partial seizures may spread via the corpus callosum to the contralateral hemisphere,
causing generalization and loss of consciousness. Division of the corpus callosum can
interrupt this spread.
Ref: Schwartz’s principle of surgery 9th edition, chapter 42.
Sample Previous Year Question on Epilepsy based on previous Year Questions of
NEET PG, USMLE,PLAB,FMGE (MCI Screening). Please visit
www.medicoapps.org for more such Quizzes
Which of the following methods of contraception should be avoided in women with
epilepsy?
A: Oral contraceptive pills
B: IUCD
C: Condoms
D: Diaphragm
Correct Ans:A
Explanation
All the antileptics (except for sodium valproate and clonazepam) have the property to
induce the enzyme complex which metabolizes the oral contraceptives. Hence it is better
avoided in patients with a history of epilepsy on medications. The other three methods of
contraception mentioned in the question have no such contraindication.
Ref: Dutta textbook of Obstetrics, 6th Edition, Page 545; Textbook of Gynaecology By Rao,
Page 183-184
Sample Previous Year Question on Epilepsy based on previous Year Questions of
NEET PG, USMLE,PLAB,FMGE (MCI Screening). Please visit
www.medicoapps.org for more such Quizzes
Which of the following drugs is not used in Juvenile Myoclonic Epilepsy (JME)?
A:
Topiramat
e
B: Zonisamide
C: Carbamezapine
D: Valproate
Correct Ans:C
Explanation
Carbamezipine, Phenytoin, Oxcarbazepine are few drugs which are capable of aggravating
Juvenile Myoclonic Epilepsy.
Ref: Epileptic Syndromes in Infancy, Childhood and Adolescence By Joseph Roger, Michelle
Bureau, Charlotte Dravet, Pierre Genton, Page 382
Sample Previous Year Question on Epilepsy based on previous Year Questions of
NEET PG, USMLE,PLAB,FMGE (MCI Screening). Please visit
www.medicoapps.org for more such Quizzes
All of the following factors are associated with a substantially greater risk of developing
epilepsy after febrile seizures, except:
A: Complex Febrile seizures
B:
Early age of
onset
C: Developmental abnormalities
D: Positive family History of Epilepsy
Correct Ans:B
Explanation
Factors increasing risk of epilepsy after febrile seizures include family history of epilepsy, a
typical seizures/complex nature of seizures and presence of a neurodevelopmental
abnormality.
Early age on onset increases the risk of recurrence of febrile convulsions.
Ref: Febrile Seizures By Tallie Z. Baram, Shlomo Shinnar, Page 63; Essential of Pediatrics By
O P Ghai, 6th Edition, Page 509
Sample Previous Year Question on Epilepsy based on previous Year Questions of
NEET PG, USMLE,PLAB,FMGE (MCI Screening). Please visit
www.medicoapps.org for more such Quizzes
An adolescent is brought to the emergency department following an episode of myoclonic
jerks at morning after waking up. His consciousness was not impaired. His EEG shows
generalized 3-4 Hz spike and slow wave complexes. Most probable diagnosis is?
A: Generalized tonic clonic seizure
B: Absent seizure
C: Temporal lobe epilepsy
D: Juvenile myoclonic epilepsy
Correct Ans:D
Explanation
Juvenile myoclonic epilepsy is a subtype of idiopathic generalized epilepsy with onset
usually between 8 and 20 years of age. Myoclonic jerks, especially in the morning, are of
variable intensity ranging from simple twitching (“flying saucer syndrome”) to
falls; consciousness is not impaired in it. It is precipitated by alcohol and sleep deprivation.
Patients will have normal intelligence. The typical interictal EEG abnormality consists of a
generalized 4- to 6-Hz spike or polyspike and slow-wave discharges lasting 1-20 seconds.
Usually, 1-3 spikes precede each slow wave.
Also know:
 In Generalized tonic clonic seizure EEG shows a normal background with
generalized epileptiform discharges such as spike or polyspike wave
complexes at 2.5 to 4 Hz.
 During absence seizures there is an abrupt onset of bilaterally
synchronous and symmetrical 3 Hz spike-wave discharge, irrespective of
whether typical absences are simple or complex.
 Ictal recordings from patients with typical temporal lobe epilepsy usually
exhibit 5-7 Hz, rhythmic, sharp theta activity, maximal in the sphenoidal
and the basal temporal electrodes on the side of seizure origin.
Ref: A-Z of Neurological Practice: A Guide to Clinical Neurology By Andrew J. Larner,
Alasdair J Coles, Neil J. Scolding, Roger A Barker, 2011, Page 368 ; Clinical
Electroencephalography by Misra,2005, Page 188
Sample Previous Year Question on Epilepsy based on previous Year Questions of
NEET PG, USMLE,PLAB,FMGE (MCI Screening). Please visit
www.medicoapps.org for more such Quizzes
Which one of the following is the characteristic feature of juvenile myoclonic epilepsy?
A: Myoclonic seizures frequently occur in morning
B:
Complete remission is
common
C: Response to anticonvulsants is poor
D: Associated absence seizures are present in majority of patients
Correct Ans:A
Explanation
Juvenile myoclonic epilepsy (JME) is a generalized seizure disorder of unknown cause that
appears in early adolescence and is usually characterized by bilateral myoclonic jerks that
may be single or repetitive. The myoclonic seizures are most frequent in the morning after
awakening and can be provoked by sleep deprivation. Consciousness is preserved unless
the myoclonus is especially severe. Many patients also experience generalized tonic-clonic
seizures, and up to one-third have absence seizures.
Although complete remission is relatively uncommon, the seizures respond well to
appropriate anticonvulsant medication. There is often a family history of epilepsy, and
genetic linkage studies suggest a polygenic cause.
Ref: Lowenstein D.H. (2012). Chapter 369. Seizures and Epilepsy. In D.L. Longo, A.S. Fauci,
D.L. Kasper, S.L. Hauser, J.L. Jameson, J. Loscalzo (Eds), Harrison's Principles of Internal
Medicine, 18e.
Sample Previous Year Question on Epilepsy based on previous Year Questions of
NEET PG, USMLE,PLAB,FMGE (MCI Screening). Please visit
www.medicoapps.org for more such Quizzes
Which of the following statements is incorrect in relation to pregnant women with epilepsy?
A: The rate of congenital malformation is increased in the offspring of women with epilepsy
B:
Seizure frequency increases in approximately 70% of
women
C: Breast feeding is safe with most anticonvulsants
D: Folic acid supplementation may reduce the risk of neural tube defect
Correct Ans:B
Explanation
Frequency of convulsions is unchanged in majority.
Ref:Textbook of Gynecology By D C Dutta, 6th Edition, Page 298; Harrison’s Principles of Internal
Medicine, 16th Edition, Page 2371
Sample Previous Year Question on Epilepsy based on previous Year Questions of
NEET PG, USMLE,PLAB,FMGE (MCI Screening). Please visit
www.medicoapps.org for more such Quizzes
Vitamin deficient is most commonly seen in a pregnant mother who is on phenytoin therapy
for epilepsy?
A: Vitamin B6
B: Vitamin B12
C: Vitamin A
D:
Folic
acid
Correct Ans:D
Explanation
Phenytoin interferes with metabolism of folate and patients taking phenytoin may become
deficient in folic acid. Phenytoin decreases absorption of folic acid and increases its
excretion.
Ref: Advanced Therapy in Epilepsy By Wheless, James W. Wheless, James Willmore, Roger
A. Brumback, 2009, Page 259.
Sample Previous Year Question on Epilepsy based on previous Year Questions of
NEET PG, USMLE,PLAB,FMGE (MCI Screening). Please visit
www.medicoapps.org for more such Quizzes
Triad of Tuberous Sclerosis includes all, except:
A: Epilepsy
B: Low intelligence
C: Hydrocephalus
D: Adenoma sebaceum
Correct Ans:C
Explanation
Diagnostic triad of Tuberous sclerosis includes epilepsy, mental retardation and adenoma
sebaceum (facial angiofibroma).
Tuberous sclerosis (TS) is an autosomal dominant disorder which result from mutations in
either the TSC1 gene encoding hamartin or the TSC2 gene encoding tuberin. Hamartin and
tubulin form a complex which negatively regulate cell growth and proliferation through
inhibition of mTOR.
Ref: Harrison’s Internal Medicine, 18th Edition, Chapter 284; The 5-Minute Neurology
Consult By D. Joanne Lynn, Page 430; Tuberous Sclerosis Complex: Genes, Clinical Features
and Therapeutics By David J. Kwiatkowsk, Page 221
Sample Previous Year Question on Epilepsy based on previous Year Questions of
NEET PG, USMLE,PLAB,FMGE (MCI Screening). Please visit
www.medicoapps.org for more such Quizzes
In pediatric patients, the risk of developing post-traumatic epilepsy is significantly
increased by:
A: Brief loss of consciousness
B: Acute intracranial hemorrhage
C: Retrograde amnesia
D: Post-traumatic vomiting
Correct Ans:B
Explanation
Late post-traumatic epilepsy is diagnosed when a seizure occurs for the first time more
than one week after a head injury. Factors that correlate with an increased risk of
developing post-traumatic epilepsy include presence of a depressed skull fracture, acute
intracranial hemorrhage, cerebral contusion, or unconsciousness lasting more than 24
hours. Because the risk of a subsequent seizure is approximately 75%, acute and chronic
treatment with anticonvulsants is indicated.
Loss of consciousness, retrograde amnesia, and vomiting are relatively common immediate
consequences of head trauma. They are usually transient and are not highly correlated with
a risk of subsequent post-traumatic seizures.
Ref: Smith W.S., English J.D., Johnston S.C. (2012). Chapter 370. Cerebrovascular Diseases.
In D.L. Longo, A.S. Fauci, D.L. Kasper, S.L. Hauser, J.L. Jameson, J. Loscalzo
(Eds),Harrison's Principles of Internal Medicine, 18e.
Sample Previous Year Question on Epilepsy based on previous Year Questions of
NEET PG, USMLE,PLAB,FMGE (MCI Screening). Please visit
www.medicoapps.org for more such Quizzes
All of the following structures pass through optic foramen, EXCEPT:
A: Optic nerve
B: Ophthalmic artery
C: Ophthalmic nerve
D: Dural mater
Correct Ans:C
Explanation
The optic foramen lies at the orbital apex and transmits the optic nerve and ophthalmic
artery. Superior orbital fissure transmits the ophthalmic nerve.
Optic foramen: Is a oval opening at the apex of the orbit. It lies between the two roots of
the lesser wings of the sphenoid bone.
It transmits
 Optic nerve
 Ophthalmic branch of internal carotid artery
 Branches of sympathetic carotid plexus
 Prolongation of optic nerve sheaths
Superior orbital fissure: The superior orbital fissure is bounded by the lesser and greater
wings of the sphenoid.
Contents:
 Superior ophthalmic vein
 Oculomotor nerve (III)
 Trochlear nerve (IV)
 Lacrimal, frontal and nasociliary branches of Ophthalmic nerve (V1)
 Abducent nerve (VI)
 Orbital branch of middle meningeal artery
 Recurrent branch of lacrimal artery
 Superior orbital vein
 Superior ophthalmic vein
Inferior orbital fissure: The greater wing of the sphenoid, the maxilla, and the palatine
bones of the orbit form the boundaries of the inferior orbital fissure.
The fissure transmits the
 Maxillary nerve and its zygomatic branch,
 Infraorbital vessels,
 Ascending branches from the sphenopalatine ganglion and a vein which
connects the inferior ophthalmic vein with the pterygoid venous plexus.
Ref: Comprehensive Manual of Ophthalmology, By Ahmed E, Page 4
Sample Previous Year Question on Foramen Part II based on previous Year
Questions of NEET PG, USMLE,PLAB,FMGE (MCI Screening). Please visit
www.medicoapps.org for more such Quizzes
Which of the following opening in the base of the skull transmits the third branch of
trigeminal nerve?
A: Foramen ovale
B: Foramen lacerum
C: Foramen magnum
D: Foramen spinosum
Correct Ans:A
Explanation
Foramen ovale is an opening at the base of the lateral pterygoid plate. It transmits the third
branch of the trigeminal nerve, the accessory meningeal artery, and occasionally the
superficial petrosal nerve.
Foramen lacerum transmits the internal carotid artery.
Foramen magnum transmits the medulla and its membranes, the spinal accessory nerves,
the vertebral arteries, and the anterior and posterior spinal arteries.
Ref: Waxman S.G. (2010). Chapter 11. Ventricles and Coverings of the Brain. In S.G.
Waxman (Ed), Clinical Neuroanatomy, 26e.
Sample Previous Year Question on Foramen Part II based on previous Year
Questions of NEET PG, USMLE,PLAB,FMGE (MCI Screening). Please visit
www.medicoapps.org for more such Quizzes
Which of the following cranial nerve travels through the jugular foramen in the base of the
skull?
A: 3rd branch of trigeminal nerve
B:
Abducens
nerve
C: Facial nerve
D: Glossopharyngeal nerve
Correct Ans:D
Explanation
Glossopharyngeal nerve travel through the middle compartment of the jugular foramen.
Jugular foramen is formed by the petrous portion of the temporal and occipital bones. It is
divided into 3 compartments. The intermediate compartment contains the
glossopharyngeal, vagus, and spinal accessory nerves. The anterior compartment contains
the inferior petrosal sinus and the posterior compartment contains the sigmoid sinus and
meningeal branches from the occipital and ascending pharyngeal arteries.
Ref: Waxman S.G. (2010). Chapter 11. Ventricles and Coverings of the Brain. In S.G.
Waxman (Ed), Clinical Neuroanatomy, 26e.
Sample Previous Year Question on Foramen Part II based on previous Year
Questions of NEET PG, USMLE,PLAB,FMGE (MCI Screening). Please visit
www.medicoapps.org for more such Quizzes
Middle meningeal artery courses along which of the following opening in the base of the
skull?
A: Foramen ovale
B: Foramen lacerum
C: Foramen spinosum
D: Foramen rotundum
Correct Ans:C
Explanation
Middle meningeal artery and vein travels through foramen spinosum.
Structures passing through openings in the skull base are:
Foramen Structures
Cribriform plate of ethmoid Olfactory nerves
Optic foramen Optic nerve, ophthalmic artery, meninges
Superior orbital fissure Oculomotor, trochlear, and abducens nerves; ophthalmic division of trigemina
Foramen rotundum Maxillary division of trigeminal nerve, small artery and vein
Foramen ovale Mandibular division of trigeminal nerve, vein
Foramen lacerum Internal carotid artery, sympathetic plexus
Foramen spinosum Middle meningeal artery and vein
Internal acoustic meatus Facial and vestibulocochlear nerves, internal auditory artery
Jugular foramen Glossopharyngeal, vagus, and spinal accessory nerves; sigmoid sinus
Hypoglossal canal Hypoglossal nerve
Foramen magnum Medulla and meninges, spinal accessory nerve, vertebral arteries, anterior an
Ref: Waxman S.G. (2010). Chapter 11. Ventricles and Coverings of the Brain. In S.G.
Waxman (Ed), Clinical Neuroanatomy, 26e.
Sample Previous Year Question on Foramen Part II based on previous Year
Questions of NEET PG, USMLE,PLAB,FMGE (MCI Screening). Please visit
www.medicoapps.org for more such Quizzes
Which of the following structure is passing through foramen rotundum?
A: Maxillary artery
B:
Maxillary
nerve
C: Middle meningeal artery
D: Spinal accessory nerve
Correct Ans:B
Explanation
Foramen rotundum located posterior to the medial end of the superior orbital fissure. The
foramen rotundum transmits the maxillary nerve (CN V-2) en route to the pterygopalatine
fossa. CN V-2 supplies the skin, teeth, and mucosa associated with the maxillary bone.
Ref: Morton D.A., Foreman K.B., Albertine K.H. (2011). Chapter 15. Scalp, Skull, and
Meninges. In D.A. Morton, K.B. Foreman, K.H. Albertine (Eds), The Big Picture: Gross
Anatomy.
Sample Previous Year Question on Foramen Part II based on previous Year
Questions of NEET PG, USMLE,PLAB,FMGE (MCI Screening). Please visit
www.medicoapps.org for more such Quizzes
Herniation through Foramen of Morgagni is seen in children. Foramen of Morgagni refers to
an opening in:
A: The brain
B: The lesser omentum
C: The skull
D: The diaphragm
Correct Ans:D
Explanation
The foramen of Morgagni occurs at the junction of the septum transversum and the anterior
thoracic wall. This anterior, central diaphragmatic defect accounts for only 2% of
diaphragmatic hernias. It may be parasternal, retrosternal, or bilateral.
The defect is closed by suturing the posterior rim of diaphragm to the posterior rectus
sheath, since there is no anterior diaphragm.
Ref: Albanese C.T., Sylvester K.G. (2010). Chapter 43. Pediatric Surgery. In G.M. Doherty
(Ed), CURRENT Diagnosis & Treatment: Surgery, 13e.
Sample Previous Year Question on Foramen Part II based on previous Year
Questions of NEET PG, USMLE,PLAB,FMGE (MCI Screening). Please visit
www.medicoapps.org for more such Quizzes
Which of the following circulatory changes normally take place in the newborn within 5
minutes after birth?
A: Closure of the ductus arteriosus
B:
Closure of the foramen
ovale
C: Increase in pulmonary artery resistance
D: All of the above
Correct Ans:B
Explanation
With the first breath of life, pulmonary arterial resistance drops dramatically. This is due to
the oxygenation of the lungs causing vasodilatation of the pulmonary vessels. Clamping of
the umbilical cord doubles peripheral resistance and causes an increase in arterial blood
pressure. As soon as right atrial, pressure drops below left atrial pressure, the foramen
ovale will close (valve-like mechanism), establishing the adult-type blood circulation.
The rising systemic and falling pulmonary artery pressure causes a flow reversal through
the ductus arteriosus from right to left to left to right within minutes of birth. Complete
closure of the ductus arteriosus appears to be due to a decline of local prostacyclin levels
and usually occurs within 24 to 48 hours after birth.
Ref: Morton D.A., Foreman K.B., Albertine K.H. (2011). Chapter 4. Heart. In D.A. Morton,
K.B. Foreman, K.H. Albertine (Eds), The Big Picture: Gross Anatomy.
Sample Previous Year Question on Foramen Part II based on previous Year
Questions of NEET PG, USMLE,PLAB,FMGE (MCI Screening). Please visit
www.medicoapps.org for more such Quizzes
Foramen of Morgagni refers to an opening in:
A: The brain
B: The lesser omentum
C: The skull
D: The diaphragm
Correct Ans:D
Explanation
Foramen of morgagni is a small triangular diaphragmatic cleft bounded medially by muscle
fibers originating from the sternum and laterally by the seventh costal cartilages. It occurs
due to lack of fusion or muscularization of the pleuroperitoneal membrane anteriorly
leading to a defect in the costosternal trigones.
Diaphragmatic hernia can occur anteriorly at the foramen of morgagni and posterolaterally
through the Bochdalek foramen or at the esophageal hiatus.
Bochdalek foramen refers to the gap between the lumbar and costal elements.
Openings in the diaphragm:
 The caval foramen is located on the right at the level of T8 within the
central tendon.
 Esophageal foramen lies at T10 level.
 The aortic opening lies behind the diaphragm at its lowest point opposite
to the T12 vertebrae.
Ref: Teaching Atlas of Chest Imaging By Mark Parker page 757, Upper Gastrointestinal
Surgery edited by John W. L. Fielding, page 118.
Sample Previous Year Question on Foramen Part II based on previous Year
Questions of NEET PG, USMLE,PLAB,FMGE (MCI Screening). Please visit
www.medicoapps.org for more such Quizzes
All of the following statements are true regarding the pudendal nerve, except:
A: It is both sensory and motor
B:
It is derived from
S2,3,4
C: It leaves the pelvis through the lesser sciatic foramen
D: It leaves through lesser sciatic foramen and enter pudendal canal
Correct Ans:C
Explanation
Pudendal nerve leaves the pelvis, to enter the gluteal region, by passing through the lower
part of the greater sciatic foramen. Pudendal nerve leaves the gluteal region by passing
through the lesser sciatic foramen and enters the pudendal canal, and by means of its
branches supplies the external anal sphincter and muscles and skin of the perineum.
Ref: Clinical anatomy for Medical Students By Richard S Snell, 6th Edition, Pages 362-3 ;
B.D.Chaurasia’s Human Anatomy, 5th Edition, Volume 2, Page 365
Sample Previous Year Question on Foramen Part II based on previous Year
Questions of NEET PG, USMLE,PLAB,FMGE (MCI Screening). Please visit
www.medicoapps.org for more such Quizzes
The boundaries of the interconnection between greater sac and lesser sac of peritoneum
known as 'Foramen of Winslow' are all, EXCEPT:
A: Caudate lobe of liver
B:
Inferior vena
cava
C: Free border of lesser omentum
D: 4th part of Duodenum
Correct Ans:D
Explanation
Interconnection between greater sac and lesser sac of peritoneum is known as Foramen of
Winslow. It has the following boundaries:
Superior boundary: Caudate lobe of liver
Anterior boundary: Free edge of lesser omentum containing common bile duct, hepatic
artery andportal vein.
Inferior boundary: First part of duodenum
Posterior boundary: Inferior vena cava and abdominal aorta
Sample Previous Year Question on Foramen Part II based on previous Year
Questions of NEET PG, USMLE,PLAB,FMGE (MCI Screening). Please visit
www.medicoapps.org for more such Quizzes
Structures passing through lesser sciatic foramen are all, except:
A: Pudendal nerve
B: Internal pudendal artery
C: Nerve to obturator externus
D: Tendon of obturator internus
Correct Ans:C
Explanation
Structures passing through lesser sciatic foramen are tendon of obturator internus muscle,
nerve to obturator internus, pudendal nerve, internal pudendal artery and vein.
Structures passing through greater sciatic foramen are:
 Piriformis
 Sciatic nerve
 Posterior cutaneous nerve of thigh
 Superior and inferior gluteal nerve
 Nerve to obturator internus and quadratus femoris
 Pudendal nerve
 Superior and inferior gluteal nerves and arteries
 Internal pudendal artery and vein
Sample Previous Year Question on Foramen Part II based on previous Year
Questions of NEET PG, USMLE,PLAB,FMGE (MCI Screening). Please visit
www.medicoapps.org for more such Quizzes
Which of the following structures form the superior border of Epiploic foramen ?
A: Quadrate lobe of liver
B: Caudate process of caudate lobe of liver
C: Porta hepatis
D: First part of the duodenum
Correct Ans:B
Explanation
In human anatomy, the omental foramen (Epiploic foramen,
foramen of Winslow, or uncommonly aditus) is the passage
of communication or foramen, between the greater sac
(general cavity of the abdomen) and the lesser sac.
Borders of Epiploic foramen:
It has the following boundaries:
 Anteriorly : free border of lesser omentum, bile duct, hepatic artery and
portal vein.
 Posteriorly : Inferior vena cava, right suprarenal gland and T12 vertebrae
 Superiorly : Caudate process of caudate lobe of liver
 Inferiorly : First part of the duodenum and horizontal part of hepatic
artery
Sample Previous Year Question on Foramen Part II based on previous Year
Questions of NEET PG, USMLE,PLAB,FMGE (MCI Screening). Please visit
www.medicoapps.org for more such Quizzes
Which of the following structure forms the superior border of Epiploic foramen?
A: Head of pancreas
B: Caudate lobe of liver
C: Lesser omentum
D: IVC
Correct Ans:B
Explanation
Caudate process of caudate lobe of the liver forms the superior border of epiploic foramen.
The right margin of the lesser sac opens into the greater sac through the opening of the
lesser sac called Epiploic foramen.
Boundaries of epiploic foramen are formed by:
Anteriorly: Free border of the lesser omentum, bile duct, hepatic artery and portal vein.
Posteriorly: Inferior vena cava
Superiorly: Caudate process of caudate lobe of the liver
Inferiorly: First part of the duodenum.
Ref: Clinical Anatomy by Regions By Richard S. Snell, 8th edn, page 208
Sample Previous Year Question on Foramen Part II based on previous Year
Questions of NEET PG, USMLE,PLAB,FMGE (MCI Screening). Please visit
www.medicoapps.org for more such Quizzes
All of the following structures forms the boundries of epiploic foramen, EXCEPT:
A: Caudate process of liver
B:
Inferior vena
cava
C: 4th part duodenum
D: Free border of lesser omentum
Correct Ans:C
Explanation
Epiploic foramen also known as foramen of winslow is a vertical slit like opening, through
which the lesser sac communicates with the greater sac.
Boundaries of the epiploic formen:
 Anteriorly: by lesser omentum containing portal vein, hepatic artery and
bile duct.
 Posteriorly: suprarenal gland inferior vena cava and T12 vertebra.
 Superiorly: caudate lobe of liver.
 Inferiorly: 1st part of duodenum and hepatic artery.
Ref: Cunningham's textbook of anatomy, By Daniel John Cunningham, Page 1239
Sample Previous Year Question on Foramen Part II based on previous Year
Questions of NEET PG, USMLE,PLAB,FMGE (MCI Screening). Please visit
www.medicoapps.org for more such Quizzes
Structure that DOES NOT pass through lesser sciatic foramen:
A: Pudendal nerve
B: Internal pudendal vessels
C: Tendon to obturator internus
D: Tendon to obturator externus
Correct Ans:D
Explanation
Lesser sciatic foramen is formed by the lesser sciatic notch and the sacrotuberous and
sacrospinous ligaments. Structures that pass through the foramen are:
 Tendon to obturatur internus
 Nerve to obturatur internus
 Pudendal nerve and
 Internal pudendal artery and vein
Ref: Snell's, Clinical Anatomy, 7th Edition, Page 602.
Sample Previous Year Question on Foramen Part II based on previous Year
Questions of NEET PG, USMLE,PLAB,FMGE (MCI Screening). Please visit
www.medicoapps.org for more such Quizzes
A 47 year old woman presents to the emergency department with cramping/colicky
abdominal pain. The current episode of pain began several hours ago, following a fatty
meal. The pain began slowly, and rose in intensity to a plateau over the course of several
hours. The patient reports that she had had several other episodes of similar pain during
the past several months, with long intervening periods of freedom from pain. On physical
examination, she is noted to have tenderness to deep palpation in the right upper quadrant
of the abdomen near the rib cage. The patient also reports that she is experiencing
shoulder/back pain at a site she identifies near the right lower scapula, but no tenderness
can be elicited during the back and shoulder examination. Following appropriate diagnostic
studies, the patient is taken to the surgical suite. During the surgery, the surgeon inserts
his fingers from right to left behind the hepatoduodenal ligament. As he does so, his fingers
enter which of the following?
A: Ampulla of Vater
B: Common bile duct
C: Epiploic foramen
D: Greater peritoneal sac
Correct Ans:C
Explanation
The space behind the stomach, hepatoduodenal ligament, and hepatogastric ligament is the
omental bursa. This space can be entered by passing through the epiploic foramen of
Winslow, as described in the question stem.
Good to know:
 The common bile duct enters the duodenum through the ampulla of Vater.
 The hepatoduodenal ligament contains the common bile duct, the portal
vein and the hepatic artery.
 The greater peritoneal sac lies anterior to the stomach and
hepatoduodenal ligament.
Ref: Morton D.A., Foreman K.B., Albertine K.H. (2011). Chapter 8. Serous Membranes of the
Abdominal Cavity. In D.A. Morton, K.B. Foreman, K.H. Albertine (Eds), The Big Picture:
Gross Anatomy.
Sample Previous Year Question on Foramen Part II based on previous Year
Questions of NEET PG, USMLE,PLAB,FMGE (MCI Screening). Please visit
www.medicoapps.org for more such Quizzes
Epiploic foramen provides communication between greater and lesser sacs. The length of
the epiploic foramen is:
A: 5 cm
B:
6
cm
C: 4 cm
D: 3 cm
Correct Ans:D
Explanation
The greater and lesser sacs communicate with each other through the epiploic foramen (of
Winslow). The foramen is about 3 cm in size and situated opposite the 12th thoracic
vertebra.
Ref: Morton D.A., Foreman K.B., Albertine K.H. (2011). Chapter 8. Serous Membranes of the
Abdominal Cavity. In D.A. Morton, K.B. Foreman, K.H. Albertine (Eds), The Big Picture:
Gross Anatomy.
Sample Previous Year Question on Foramen Part II based on previous Year
Questions of NEET PG, USMLE,PLAB,FMGE (MCI Screening). Please visit
www.medicoapps.org for more such Quizzes
Where is the ‘Foramen of Winslow’ located anatomically?
A: Between greater and lesser sac
B:
At hilum of
liver
C: Transverse cervical ligament of uterus
D: Pouch of Douglas
Correct Ans:A
Explanation
The lesser omentum attaches between the liver, stomach, and proximal portion of the duodenum. It
forms a sac known as the omental bursa, which forms a subdivision of the peritoneal cavity known
as the lesser sac. The greater sac is the remaining part of the peritoneal cavity.
The greater and lesser sacs communicate with each other through the epiploic foramen (of
Winslow).
Ref: Morton D.A., Foreman K.B., Albertine K.H. (2011). Chapter 8. Serous Membranes of the
Abdominal Cavity. In D.A. Morton, K.B. Foreman, K.H. Albertine (Eds), The Big Picture:
Gross Anatomy.
Sample Previous Year Question on Foramen Part II based on previous Year
Questions of NEET PG, USMLE,PLAB,FMGE (MCI Screening). Please visit
www.medicoapps.org for more such Quizzes
Which of the following is the anatomical location of foramen of winslow?
A: Between greater and lesser sac
B:
At hilum of
liver
C: Transverse cervical ligament of uterus
D: Pouch of Douglas
Correct Ans:A
Explanation
The lesser omentum forms a sac known as the omental bursa, which forms a subdivision of
the peritoneal cavity known as the lesser sac. The greater sac is the remaining part of the
peritoneal cavity. The greater and lesser sacs communicate with each other through
the epiploic foramen (of Winslow).
Also know:
The omentum refers to modified mesenteries associated with the stomach and liver.
 Greater omentum. An apron-like fold of mesentery that attaches between
the transverse colon to the greater curvature of the stomach.
 Lesser omentum. Mesentery that attaches between the liver, stomach, and
proximal portion of the duodenum. As a result, the lesser omentum is also
referred to as thehepatogastric ligament and hepatoduodenal ligament.
Ref: Morton D.A., Foreman K.B., Albertine K.H. (2011). Chapter 8. Serous Membranes of the
Abdominal Cavity. In D.A. Morton, K.B. Foreman, K.H. Albertine (Eds), The Big Picture:
Gross Anatomy.
Sample Previous Year Question on Foramen Part II based on previous Year
Questions of NEET PG, USMLE,PLAB,FMGE (MCI Screening). Please visit
www.medicoapps.org for more such Quizzes
Where is the anatomical location of Foramen of Winslow?
A: Between greater and lesser sac
B:
At hilum of
liver
C: Transverse cervical ligament of uterus
D: Pouch of Douglas
Correct Ans:A
Explanation
The lesser omentum forms a sac known as the omental bursa, which forms a subdivision of
the peritoneal cavity known as the lesser sac. The greater sac is the remaining part of the
peritoneal cavity. The greater and lesser sacs communicate with each other through
the epiploic foramen (of Winslow).
Boundaries of epiploic foramen:
 Anteriorly: The free border of the lesser omentum, containing the bile
duct, the hepatic artery, and the portal vein
 Posteriorly: Inferior vena cava
 Superiorly: The caudate process of the caudate lobe of the liver
 Inferiorly: The first part of the duodenum
Ref: Morton D.A., Foreman K.B., Albertine K.H. (2011). Chapter 8. Serous Membranes of the
Abdominal Cavity. In D.A. Morton, K.B. Foreman, K.H. Albertine (Eds), The Big Picture:
Gross Anatomy.
Sample Previous Year Question on Foramen Part II based on previous Year
Questions of NEET PG, USMLE,PLAB,FMGE (MCI Screening). Please visit
www.medicoapps.org for more such Quizzes
The posterior relation of epiploic foramen is:
A: Hepatic artery
B: Inferior vena cava
C: Common bile duct
D: Portal vein
Correct Ans:B
Explanation
Epiploic foramen or aditus to lesser sac communicates lesser sac to greater sac. It is
bounded posteriorly by inferior vena cava, suprarenal gland and T12 vertebra.
Epiploic foramen: also known as foramen of winslow is a vertical slit like opening, through
which the lesser sac communicates with the greater sac. It is situated at the upper wall of
lesser sac at the level of T12.
Boundaries of the epiploic foramen:
 Anteriorly: right free margin of lesser omentum containing portal vein,
hepatic artery and bile duct
 Posteriorly: suprarenal gland, inferior vena cava and T12 vertebra
 Superiorly: caudate lobe of liver
 Inferiorly: 1st part of duodenum and hepatic artery
Ref: Cunningham's textbook of anatomy, By Daniel John Cunningham, Page 1239
Sample Previous Year Question on Foramen Part III based on previous Year
Questions of NEET PG, USMLE,PLAB,FMGE (MCI Screening). Please visit
www.medicoapps.org for more such Quizzes
All the following structures form boundaries of Epiploic foramen, EXCEPT:
A: Portal vein
B: Inferior vena cava
C: Quadrate lobe of liver
D: First part of duodenum
Correct Ans:C
Explanation
The right margin of the lesser sac opens into the greater sac through the epiploic foramen.
The superior boundary of the foramen is foramen by caudate lobe of the liver.
Structures forming boundaries of the epiploic foramen are:
Anteriorly: free border of lesser omentum, bile duct, hepatic artery and portal vein.
Posteriorly: Inferior vena cava
Superiorly: Caudate process of caudate lobe of liver
Inferiorly: First part of duodenum
Ref: Clinical Anatomy by Regions By Richard S. Snell page 208.
Sample Previous Year Question on Foramen Part III based on previous Year
Questions of NEET PG, USMLE,PLAB,FMGE (MCI Screening). Please visit
www.medicoapps.org for more such Quizzes
All of the following structures pass through the greater sciatic foramen, EXCEPT:
A: Piriformis muscle
B:
Pudendal
nerve
C: Inferior gluteal vessel
D: Internal pudendal vessel
Correct Ans:B
Explanation
Pudendal nerve courses through the lesser sciatic foramen.
Structures passing through the lesser sciatic foramen are:
 Pudendal nerve
 Internal pudendal vessels
 Obturator internus tendon
Structures passing through the greater sciatic foramen are:
 Sciatic nerve
 Piriformis muscle
 Internal pudendal and inferior gluteal vessels
 Other branches of the sacral nerve plexus
Ref: Hoffman B.L., Schorge J.O., Schaffer J.I., Halvorson L.M., Bradshaw K.D., Cunningham
F.G., Calver L.E. (2012). Chapter 38. Anatomy.
Sample Previous Year Question on Foramen Part III based on previous Year
Questions of NEET PG, USMLE,PLAB,FMGE (MCI Screening). Please visit
www.medicoapps.org for more such Quizzes
Which of the following is TRUE about ‘Foramen of Winslow’?
A: It is seen between greater and lesser sac
B: Anteriorly bound by 2nd part of duodenum
C: Seen in transverse cervical ligament of uterus
D: Posteriorly bound by pouch of Douglas
Correct Ans:A
Explanation
The epiploic foramen of Winslow or the aditus to the lesser sac is the only communication
between greater and lesser sac. It is bounded,
 Anteriorly by hepatoduodenal ligament
 Posteriorly by inferior venacava
 Superiorly by caudate lobe of liver
 Inferiorly by first part of duodenum
Ref: Last's Anatomy: Regional and Applied By Chummy S. Sinnatamby, 2011, Chapter 5.
Sample Previous Year Question on Foramen Part III based on previous Year
Questions of NEET PG, USMLE,PLAB,FMGE (MCI Screening). Please visit
www.medicoapps.org for more such Quizzes
All of the following passing through lesser sciatic foramen, except:
A: Pudendal nerve
B: Internal pudendal vessels
C: Nerve to obturator internus
D: Inferior gluteal vessels
Correct Ans:D
Explanation
The lesser sciatic foramen is formed by the lesser sciatic notch of the pelvic bone, the
ischial,the sacrospinous ligament and the sacrotuberous ligament.
It transmits the following structures:
 The tendon of the Obturator internus
 Internal pudendal vessels
 Pudendal nerve
 Nerve to the obturator internus
Ref: Morton D.A., Foreman K.B., Albertine K.H. (2011).
Chapter 12. Pelvis and Perineum. In D.A. Morton, K.B.
Foreman, K.H. Albertine (Eds), The Big Picture: Gross
Anatomy.
Sample Previous Year Question on Foramen Part III based on previous Year
Questions of NEET PG, USMLE,PLAB,FMGE (MCI Screening). Please visit
www.medicoapps.org for more such Quizzes
All of the following structures passes through lesser sciatic foramen, EXCEPT:
A: Inferior gluteal vessels
B: Internal pudendal vessels
C: Pudendal nerve
D: Nerve to obturator internus
Correct Ans:A
Explanation
Structures passing through the lesser sciatic foramen are tendon of obturator internus,
internal pudendal vessels and pudendal nerve.
Structures passing through greater sciatic foramen are:
 Piriformis muscle
 Superior and inferior gluteal vessel
 Internal pudendal vessel
 Pudendal nerve
 Sciatic nerve
 Posterior femoral cutaneous nerve
 Nerve to obturator internus
 Quadratus femoris
Structures passing through both greater and lesser sciatic foramen:
 Pudendal nerve
 Internal pudendal vessels
 Nerve to obturator internus
Ref: BRS Gross Anatomy By Kyung Won Chung, page 97
Sample Previous Year Question on Foramen Part III based on previous Year
Questions of NEET PG, USMLE,PLAB,FMGE (MCI Screening). Please visit
www.medicoapps.org for more such Quizzes
The middle meningeal artery enters the cranium through the:
A: Foramen spinosum
B: Foramen rotundum
C: Foramen magnum
D: Foramen ovale
Correct Ans:A
Explanation
The middle meningeal artery is the largest of the arteries supplying the dura mater. It is a
branch of the maxillary artery and enters the cranium through the foramen spinosum.
Ref: Gray's Basic Anatomy International Ed, page 429
Sample Previous Year Question on Foramen Part III based on previous Year
Questions of NEET PG, USMLE,PLAB,FMGE (MCI Screening). Please visit
www.medicoapps.org for more such Quizzes
Which of the following foramen is responsible for the drainage of CSF from lateral to third
ventricle?
A: Foramen of Monro
B: Foramen of Luschka
C: Foramen of Magendie
D: Cerebral aqueduct
Correct Ans:A
Explanation
The two interventricular foramens, or foramens of Monro, are apertures between the
column of the fornix and the anterior end of the thalamus. The two lateral ventricles
communicate with the third ventricle through these foramens and drainage of CSF occurs.
The lateral aperture (foramen of Luschka) is the opening of the lateral recess into the
subarachnoid space near the flocculus of the cerebellum. A tuft of choroid plexus is
commonly present in the aperture and partly obstructs the flow of CSF from the fourth
ventricle to the subarachnoid space.
The medial aperture (foramen of Magendie) is an opening in the caudal portion of the roof
of the ventricle. Most of the outflow of CSF from the fourth ventricle passes through this
aperture.
The cerebral aqueduct is a narrow, curved channel running from the posterior third
ventricle into the fourth.
Ref: Waxman S.G. (2010). Chapter 11. Ventricles and Coverings of the Brain. In S.G.
Waxman (Ed), Clinical Neuroanatomy, 26e.
Sample Previous Year Question on Foramen Part III based on previous Year
Questions of NEET PG, USMLE,PLAB,FMGE (MCI Screening). Please visit
www.medicoapps.org for more such Quizzes
A 17-year-old male is examined by a physician, who notes a mass at the back of the young
man's tongue. The physician biopsies the mass, and the pathology report comes back with a
diagnosis of normal thyroid tissue. The occasional presence of such tissue at the back of the
tongue is related to the embryonic origin of the thyroid near which of the following
structures?
A: First pharyngeal pouch
B:
Foramen
cecum
C: Nasolacrimal duct
D: Second pharyngeal arch
Correct Ans:B
Explanation
The thyroid gland originates as a mass of endodermal tissue near the foramen cecum,
which is near the tuberculum impar (which becomes the central part of the tongue). During
development, the thyroid descends in front of the pharynx, maintaining a connection to the
tongue via the thyroglossal duct.
Usually, the thyroglossal duct disappears. Uncommonly, residual ectopic thyroid tissue can
be left anywhere along the path, including at the back of the tongue. (In rare patients, all of
the thyroid tissue remains at this site, forming a mass that should not be excised, for
obvious reasons!).
The first pharyngeal pouch develops into the middle ear and eustachian tube. The
nasolacrimal ducts connect the eyes to the mouth. The second pharyngeal arch develops
into many muscles of the face and styloid process of the temporal bone.
Sample Previous Year Question on Foramen Part III based on previous Year
Questions of NEET PG, USMLE,PLAB,FMGE (MCI Screening). Please visit
www.medicoapps.org for more such Quizzes
A 17-year-old male is examined by a physician, who notes a mass at the back of the young
man's tongue. The physician biopsies the mass, and the pathology report comes back with a
diagnosis of normal thyroid tissue. The occasional presence of such tissue at the back of the
tongue is related to the embryonic origin of the thyroid near which of the following
structures?
A: First pharyngeal pouch
B:
Foramen
cecum
C: Nasolacrimal duct
D: Second pharyngeal arch
Correct Ans:B
Explanation
The thyroid gland originates as a mass of endodermal tissue near the foramen cecum,
which is near the tuberculum impar (which becomes the central part of the tongue). During
development, the thyroid descends in front of the pharynx, maintaining a connection to the
tongue via the thyroglossal duct.
Usually, the thyroglossal duct disappears. Uncommonly, residual ectopic thyroid tissue can
be left anywhere along the path, including at the back of the tongue. (In rare patients, all of
the thyroid tissue remains at this site, forming a mass that should not be excised, for
obvious reasons!).
The first pharyngeal pouch develops into the middle ear and eustachian tube. The
nasolacrimal ducts connect the eyes to the mouth. The second pharyngeal arch develops
into many muscles of the face and styloid process of the temporal bone.
Sample Previous Year Question on Foramen Part III based on previous Year
Questions of NEET PG, USMLE,PLAB,FMGE (MCI Screening). Please visit
www.medicoapps.org for more such Quizzes
Which gland is derived from foramen caecum?
A: Pituitary
B:
Thyroi
d
C: Thymus
D: Parathyroid
Correct Ans:B
Explanation
The site of origin of the thyroglossal duct is the foramen caecum. It grows down in the
midline into the neck. Its tip soon bifurcates. Proliferation of the cells of this bifid end gives
rise to the two lobes of the thyroid.
Ref: Textbook of Human Embryology, 6th Ed Page 119.
Sample Previous Year Question on Foramen Part III based on previous Year
Questions of NEET PG, USMLE,PLAB,FMGE (MCI Screening). Please visit
www.medicoapps.org for more such Quizzes
All of the following characteristics differentiate a typical cervical vertebrae from a thoracic
vertebrae except
A: Mas a triangular vertebral canal
B:
Has foramen
transversarium
C: Superior articular facet is directed backwards & upwards.
D: Has a large vertebral body
Correct Ans:D
Explanation
The transverse process of cervical vertebrae is pierced by a foramen called the foramen
transversarium.The laminae of cervical vertebrae are long(transversely)and narrow
(vertically).The spinous processes are short and bifid in a typical cervical vertebra.
The vertebral bodies progressively increase in size from above downwards.They are
therefore smallest in the cervical vertebrae and largest in the lumbar vertebrae.
Ref: Textbook of Anatomy with Colour Atlas by Inderbir Singh By Inderbir Singh Page 85
Sample Previous Year Question on Foramen Part III based on previous Year
Questions of NEET PG, USMLE,PLAB,FMGE (MCI Screening). Please visit
www.medicoapps.org for more such Quizzes
Cervical vertebrae can be differentiated from thoracic vertebrae on the basis of which of the
following features?
A: Triangular body
B: Presence of foramen transversarium
C: Superior articular facet which is directed backwards and sidewards
D: Presence of a large vertebral body
Correct Ans:B
Explanation
Foramen transversarium is a characteristic feature of
cervical vertebrae. Foramen transversarium of transverse
process of cervical vertebrae transmits:
 Vertebral artery,
 Vertebral veins,
 Branch from the inferior cervical sympathetic ganglion.
Thoracic vertebrae is chracterised by the presence of costal facets in the body.
Lumbar vertebrae has:
 Kidney shaped body
 Long thin transverse process
 Spine which is short, quadrilateral and horizontal
 Presence of mamillary body and accessary mamillary process
 Absence of foramen transvorsorium and costal facets.
Sample Previous Year Question on Foramen Part III based on previous Year
Questions of NEET PG, USMLE,PLAB,FMGE (MCI Screening). Please visit
www.medicoapps.org for more such Quizzes
All of the following characteristics differentiate a typical cervical vertebrae from a thoracic
vertebrae, EXCEPT:
A: Has a triangular vertebral canal
B:
Has foramen
transversarium
C: Superior articular facet is directed backwards and upwards
D: Has a large vertebral body
Correct Ans:D
Explanation
Cervical vertebrae has a small body. Its transverse diameter is greater than the
anteroposterior length. Outline of thoracic vertebral body when viewed from above is heart
shaped.
Features of a typical cervical vertebrae:
 Each transverse process has foramen transversarium.
 The spines are short and bifid.
 Vertebral foramen is large and triangular.
Features of thoracic vertebrae:
 Presence of costal facets on each side of the body for articulation with
ribs.
 Their spinous process are long, slender, and directed downwards.
 Vertebral foramina is circular.
Ref: TEXTBOOK of ANATOMY and PHYSIOLOGY for NURSES ASHALATA By Jaypee Brothers,
page 72.
Sample Previous Year Question on Foramen Part III based on previous Year
Questions of NEET PG, USMLE,PLAB,FMGE (MCI Screening). Please visit
www.medicoapps.org for more such Quizzes
Which of the following is the location of the mental foramen?
A: Between the premolar of mandible
B:
First molar of
mandible
C: Canine of mandible
D: Canine of maxilla
Correct Ans:A
Explanation
The mental foramen lies below the interval between the premolar teeth. The mental nerve
exits the mandible through the mental foramen and supplies the lower lip and chin. The
mental nerve is palpable and sometimes visible through the oral mucosa adjacent to the
roots of the premolar tooth.
Ref: Gray's Basic Anatomy By Richard Drake, A. Wayne Vogl, Adam W. M. Mitchell, 2012,
Page 504.
Sample Previous Year Question on Foramen Part III based on previous Year
Questions of NEET PG, USMLE,PLAB,FMGE (MCI Screening). Please visit
www.medicoapps.org for more such Quizzes

More Related Content

What's hot

Previous year question on carcinma urinary bladder based on neet pg, usmle, p...
Previous year question on carcinma urinary bladder based on neet pg, usmle, p...Previous year question on carcinma urinary bladder based on neet pg, usmle, p...
Previous year question on carcinma urinary bladder based on neet pg, usmle, p...Abhishek Gupta
 
Previous year question on poisoninig part i and ii based on neet pg, usmle, p...
Previous year question on poisoninig part i and ii based on neet pg, usmle, p...Previous year question on poisoninig part i and ii based on neet pg, usmle, p...
Previous year question on poisoninig part i and ii based on neet pg, usmle, p...Abhishek Gupta
 
Previous year question on anorexia based on neet pg, usmle, plab and fmge or ...
Previous year question on anorexia based on neet pg, usmle, plab and fmge or ...Previous year question on anorexia based on neet pg, usmle, plab and fmge or ...
Previous year question on anorexia based on neet pg, usmle, plab and fmge or ...Abhishek Gupta
 
Previous year question on aspermia based on neet pg, usmle, plab and fmge or ...
Previous year question on aspermia based on neet pg, usmle, plab and fmge or ...Previous year question on aspermia based on neet pg, usmle, plab and fmge or ...
Previous year question on aspermia based on neet pg, usmle, plab and fmge or ...Abhishek Gupta
 
Previous year question on mumps based on neet pg, usmle, plab and fmge or mci...
Previous year question on mumps based on neet pg, usmle, plab and fmge or mci...Previous year question on mumps based on neet pg, usmle, plab and fmge or mci...
Previous year question on mumps based on neet pg, usmle, plab and fmge or mci...Abhishek Gupta
 
Previous year question on polio based on neet pg, usmle, plab and fmge or mci...
Previous year question on polio based on neet pg, usmle, plab and fmge or mci...Previous year question on polio based on neet pg, usmle, plab and fmge or mci...
Previous year question on polio based on neet pg, usmle, plab and fmge or mci...Abhishek Gupta
 
Previous year question on rabies based on neet pg, usmle, plab and fmge or mc...
Previous year question on rabies based on neet pg, usmle, plab and fmge or mc...Previous year question on rabies based on neet pg, usmle, plab and fmge or mc...
Previous year question on rabies based on neet pg, usmle, plab and fmge or mc...Medico Apps
 
Epilepsy questions for neet pg, usmle, plab , fmge (mci screening)
Epilepsy questions for neet pg, usmle, plab , fmge (mci screening)Epilepsy questions for neet pg, usmle, plab , fmge (mci screening)
Epilepsy questions for neet pg, usmle, plab , fmge (mci screening)Medico Apps
 
Pain leaflet brittle bone disease
Pain leaflet brittle bone diseasePain leaflet brittle bone disease
Pain leaflet brittle bone diseaseAlison Stevens
 
Intra Cranial Stem Cell Transplant For Npc.Ppt 2
Intra Cranial Stem Cell Transplant For Npc.Ppt 2Intra Cranial Stem Cell Transplant For Npc.Ppt 2
Intra Cranial Stem Cell Transplant For Npc.Ppt 2Duriya Lakdawala
 
Impact Of Breast Feeding On Admission For Pneumonia During Postneonatal Perio...
Impact Of Breast Feeding On Admission For Pneumonia During Postneonatal Perio...Impact Of Breast Feeding On Admission For Pneumonia During Postneonatal Perio...
Impact Of Breast Feeding On Admission For Pneumonia During Postneonatal Perio...Biblioteca Virtual
 
Angela Poster Draft.JB
Angela Poster Draft.JBAngela Poster Draft.JB
Angela Poster Draft.JBAngela Luo
 

What's hot (19)

Previous year question on carcinma urinary bladder based on neet pg, usmle, p...
Previous year question on carcinma urinary bladder based on neet pg, usmle, p...Previous year question on carcinma urinary bladder based on neet pg, usmle, p...
Previous year question on carcinma urinary bladder based on neet pg, usmle, p...
 
Previous year question on poisoninig part i and ii based on neet pg, usmle, p...
Previous year question on poisoninig part i and ii based on neet pg, usmle, p...Previous year question on poisoninig part i and ii based on neet pg, usmle, p...
Previous year question on poisoninig part i and ii based on neet pg, usmle, p...
 
Previous year question on anorexia based on neet pg, usmle, plab and fmge or ...
Previous year question on anorexia based on neet pg, usmle, plab and fmge or ...Previous year question on anorexia based on neet pg, usmle, plab and fmge or ...
Previous year question on anorexia based on neet pg, usmle, plab and fmge or ...
 
Previous year question on aspermia based on neet pg, usmle, plab and fmge or ...
Previous year question on aspermia based on neet pg, usmle, plab and fmge or ...Previous year question on aspermia based on neet pg, usmle, plab and fmge or ...
Previous year question on aspermia based on neet pg, usmle, plab and fmge or ...
 
Previous year question on mumps based on neet pg, usmle, plab and fmge or mci...
Previous year question on mumps based on neet pg, usmle, plab and fmge or mci...Previous year question on mumps based on neet pg, usmle, plab and fmge or mci...
Previous year question on mumps based on neet pg, usmle, plab and fmge or mci...
 
Previous year question on polio based on neet pg, usmle, plab and fmge or mci...
Previous year question on polio based on neet pg, usmle, plab and fmge or mci...Previous year question on polio based on neet pg, usmle, plab and fmge or mci...
Previous year question on polio based on neet pg, usmle, plab and fmge or mci...
 
Previous year question on rabies based on neet pg, usmle, plab and fmge or mc...
Previous year question on rabies based on neet pg, usmle, plab and fmge or mc...Previous year question on rabies based on neet pg, usmle, plab and fmge or mc...
Previous year question on rabies based on neet pg, usmle, plab and fmge or mc...
 
Epilepsy questions for neet pg, usmle, plab , fmge (mci screening)
Epilepsy questions for neet pg, usmle, plab , fmge (mci screening)Epilepsy questions for neet pg, usmle, plab , fmge (mci screening)
Epilepsy questions for neet pg, usmle, plab , fmge (mci screening)
 
Pain leaflet brittle bone disease
Pain leaflet brittle bone diseasePain leaflet brittle bone disease
Pain leaflet brittle bone disease
 
Innis Paper Sbs
Innis Paper SbsInnis Paper Sbs
Innis Paper Sbs
 
Autoimmunity and non
Autoimmunity and nonAutoimmunity and non
Autoimmunity and non
 
Innis
InnisInnis
Innis
 
ALL (PCNA)
ALL (PCNA)ALL (PCNA)
ALL (PCNA)
 
Intra Cranial Stem Cell Transplant For Npc.Ppt 2
Intra Cranial Stem Cell Transplant For Npc.Ppt 2Intra Cranial Stem Cell Transplant For Npc.Ppt 2
Intra Cranial Stem Cell Transplant For Npc.Ppt 2
 
Impact Of Breast Feeding On Admission For Pneumonia During Postneonatal Perio...
Impact Of Breast Feeding On Admission For Pneumonia During Postneonatal Perio...Impact Of Breast Feeding On Admission For Pneumonia During Postneonatal Perio...
Impact Of Breast Feeding On Admission For Pneumonia During Postneonatal Perio...
 
Pathology of CNS Tumors - Quiz
Pathology of CNS Tumors - QuizPathology of CNS Tumors - Quiz
Pathology of CNS Tumors - Quiz
 
DIGITAL NEURORADIOLOGY
DIGITAL NEURORADIOLOGYDIGITAL NEURORADIOLOGY
DIGITAL NEURORADIOLOGY
 
Genetics and health
Genetics and healthGenetics and health
Genetics and health
 
Angela Poster Draft.JB
Angela Poster Draft.JBAngela Poster Draft.JB
Angela Poster Draft.JB
 

Similar to 22-year-old patient's pre-anaesthetic review for epilepsy history

Genetic epilepsy Ahmed.pptx
Genetic epilepsy Ahmed.pptxGenetic epilepsy Ahmed.pptx
Genetic epilepsy Ahmed.pptxAhmed Moharram
 
FNU Pathophysiology Quiz 4.pdf
FNU Pathophysiology Quiz 4.pdfFNU Pathophysiology Quiz 4.pdf
FNU Pathophysiology Quiz 4.pdfsdfghj21
 
Premier Medillectuals - Mains
Premier Medillectuals - MainsPremier Medillectuals - Mains
Premier Medillectuals - MainsDhananjay Bansal
 
recent seminar topic for m.pharm
recent seminar topic for m.pharmrecent seminar topic for m.pharm
recent seminar topic for m.pharmnilesh1208
 
Pediatric neurology for ug part 1
Pediatric neurology for ug part 1Pediatric neurology for ug part 1
Pediatric neurology for ug part 1Hari Meshram
 
Ipoglicemia da iperinsulinemia
Ipoglicemia da iperinsulinemiaIpoglicemia da iperinsulinemia
Ipoglicemia da iperinsulinemiaMerqurio
 
2nd Pediatric On Squares Pediatric Board Review.pdf
2nd Pediatric On Squares Pediatric Board Review.pdf2nd Pediatric On Squares Pediatric Board Review.pdf
2nd Pediatric On Squares Pediatric Board Review.pdfMEWBORG
 
Previous year question on hepatomegaly based on neet pg, usmle, plab and fmge...
Previous year question on hepatomegaly based on neet pg, usmle, plab and fmge...Previous year question on hepatomegaly based on neet pg, usmle, plab and fmge...
Previous year question on hepatomegaly based on neet pg, usmle, plab and fmge...Medico Apps
 
PRES, the most uncommon side effect of one of the commonest chemotherapy regi...
PRES, the most uncommon side effect of one of the commonest chemotherapy regi...PRES, the most uncommon side effect of one of the commonest chemotherapy regi...
PRES, the most uncommon side effect of one of the commonest chemotherapy regi...Apollo Hospitals
 
UWSA 1 2021 For USMLE Step 1 Exam
UWSA 1 2021 For USMLE Step 1 ExamUWSA 1 2021 For USMLE Step 1 Exam
UWSA 1 2021 For USMLE Step 1 Examusmlematerialsnet
 
Previous year question on menorrhagia based on neet pg, usmle, plab and fmge...
Previous year question on menorrhagia  based on neet pg, usmle, plab and fmge...Previous year question on menorrhagia  based on neet pg, usmle, plab and fmge...
Previous year question on menorrhagia based on neet pg, usmle, plab and fmge...Abhishek Gupta
 
Previous year question on tonsils based on neet pg, usmle, plab and fmge or m...
Previous year question on tonsils based on neet pg, usmle, plab and fmge or m...Previous year question on tonsils based on neet pg, usmle, plab and fmge or m...
Previous year question on tonsils based on neet pg, usmle, plab and fmge or m...Abhishek Gupta
 
wilkinsdeliriumelderly_101889_284_38753_v1.ppt
wilkinsdeliriumelderly_101889_284_38753_v1.pptwilkinsdeliriumelderly_101889_284_38753_v1.ppt
wilkinsdeliriumelderly_101889_284_38753_v1.pptSumairaKanwal19
 
A guideline for discontinuing antiepileptic drugs in seizure-free patients – ...
A guideline for discontinuing antiepileptic drugs in seizure-free patients – ...A guideline for discontinuing antiepileptic drugs in seizure-free patients – ...
A guideline for discontinuing antiepileptic drugs in seizure-free patients – ...Dr. Rafael Higashi
 

Similar to 22-year-old patient's pre-anaesthetic review for epilepsy history (20)

Genetic epilepsy Ahmed.pptx
Genetic epilepsy Ahmed.pptxGenetic epilepsy Ahmed.pptx
Genetic epilepsy Ahmed.pptx
 
FNU Pathophysiology Quiz 4.pdf
FNU Pathophysiology Quiz 4.pdfFNU Pathophysiology Quiz 4.pdf
FNU Pathophysiology Quiz 4.pdf
 
Premier Medillectuals - Mains
Premier Medillectuals - MainsPremier Medillectuals - Mains
Premier Medillectuals - Mains
 
Premier Medillectuals :- Mains
Premier Medillectuals :- MainsPremier Medillectuals :- Mains
Premier Medillectuals :- Mains
 
recent seminar topic for m.pharm
recent seminar topic for m.pharmrecent seminar topic for m.pharm
recent seminar topic for m.pharm
 
Pediatric neurology for ug part 1
Pediatric neurology for ug part 1Pediatric neurology for ug part 1
Pediatric neurology for ug part 1
 
Epilepsy CME Eldoret 12th March 2015
Epilepsy CME Eldoret 12th March 2015Epilepsy CME Eldoret 12th March 2015
Epilepsy CME Eldoret 12th March 2015
 
Myasthenia gravis in pregnancy
Myasthenia gravis in pregnancyMyasthenia gravis in pregnancy
Myasthenia gravis in pregnancy
 
Ipoglicemia da iperinsulinemia
Ipoglicemia da iperinsulinemiaIpoglicemia da iperinsulinemia
Ipoglicemia da iperinsulinemia
 
2nd Pediatric On Squares Pediatric Board Review.pdf
2nd Pediatric On Squares Pediatric Board Review.pdf2nd Pediatric On Squares Pediatric Board Review.pdf
2nd Pediatric On Squares Pediatric Board Review.pdf
 
Previous year question on hepatomegaly based on neet pg, usmle, plab and fmge...
Previous year question on hepatomegaly based on neet pg, usmle, plab and fmge...Previous year question on hepatomegaly based on neet pg, usmle, plab and fmge...
Previous year question on hepatomegaly based on neet pg, usmle, plab and fmge...
 
PRES, the most uncommon side effect of one of the commonest chemotherapy regi...
PRES, the most uncommon side effect of one of the commonest chemotherapy regi...PRES, the most uncommon side effect of one of the commonest chemotherapy regi...
PRES, the most uncommon side effect of one of the commonest chemotherapy regi...
 
UWSA 1 2021 For USMLE Step 1 Exam
UWSA 1 2021 For USMLE Step 1 ExamUWSA 1 2021 For USMLE Step 1 Exam
UWSA 1 2021 For USMLE Step 1 Exam
 
Previous year question on menorrhagia based on neet pg, usmle, plab and fmge...
Previous year question on menorrhagia  based on neet pg, usmle, plab and fmge...Previous year question on menorrhagia  based on neet pg, usmle, plab and fmge...
Previous year question on menorrhagia based on neet pg, usmle, plab and fmge...
 
Previous year question on tonsils based on neet pg, usmle, plab and fmge or m...
Previous year question on tonsils based on neet pg, usmle, plab and fmge or m...Previous year question on tonsils based on neet pg, usmle, plab and fmge or m...
Previous year question on tonsils based on neet pg, usmle, plab and fmge or m...
 
wilkinsdeliriumelderly_101889_284_38753_v1.ppt
wilkinsdeliriumelderly_101889_284_38753_v1.pptwilkinsdeliriumelderly_101889_284_38753_v1.ppt
wilkinsdeliriumelderly_101889_284_38753_v1.ppt
 
West syndrome
West syndrome West syndrome
West syndrome
 
A guideline for discontinuing antiepileptic drugs in seizure-free patients – ...
A guideline for discontinuing antiepileptic drugs in seizure-free patients – ...A guideline for discontinuing antiepileptic drugs in seizure-free patients – ...
A guideline for discontinuing antiepileptic drugs in seizure-free patients – ...
 
Myasthenia gravis
Myasthenia gravisMyasthenia gravis
Myasthenia gravis
 
Epilepsy
EpilepsyEpilepsy
Epilepsy
 

More from Abhishek Gupta

Medico apps resources on clinical signs
Medico apps resources on clinical signsMedico apps resources on clinical signs
Medico apps resources on clinical signsAbhishek Gupta
 
Previous year question on staining based on neet pg, usmle, plab and fmge or ...
Previous year question on staining based on neet pg, usmle, plab and fmge or ...Previous year question on staining based on neet pg, usmle, plab and fmge or ...
Previous year question on staining based on neet pg, usmle, plab and fmge or ...Abhishek Gupta
 
Compilation of syndromes for last minute revision tips for neet pg, usmle , p...
Compilation of syndromes for last minute revision tips for neet pg, usmle , p...Compilation of syndromes for last minute revision tips for neet pg, usmle , p...
Compilation of syndromes for last minute revision tips for neet pg, usmle , p...Abhishek Gupta
 
Previous year question on pneumothorax based on neet pg, usmle, plab and fmge...
Previous year question on pneumothorax based on neet pg, usmle, plab and fmge...Previous year question on pneumothorax based on neet pg, usmle, plab and fmge...
Previous year question on pneumothorax based on neet pg, usmle, plab and fmge...Abhishek Gupta
 
Previous year question on schwannoma based on neet pg, usmle, plab and fmge o...
Previous year question on schwannoma based on neet pg, usmle, plab and fmge o...Previous year question on schwannoma based on neet pg, usmle, plab and fmge o...
Previous year question on schwannoma based on neet pg, usmle, plab and fmge o...Abhishek Gupta
 
Previous year question on lichen planus based on neet pg, usmle, plab and fmg...
Previous year question on lichen planus based on neet pg, usmle, plab and fmg...Previous year question on lichen planus based on neet pg, usmle, plab and fmg...
Previous year question on lichen planus based on neet pg, usmle, plab and fmg...Abhishek Gupta
 
Previous year question on lichen planus based on neet pg, usmle, plab and fmg...
Previous year question on lichen planus based on neet pg, usmle, plab and fmg...Previous year question on lichen planus based on neet pg, usmle, plab and fmg...
Previous year question on lichen planus based on neet pg, usmle, plab and fmg...Abhishek Gupta
 
Previous year question on otosclerosis from ent based on neet pg, usmle, plab...
Previous year question on otosclerosis from ent based on neet pg, usmle, plab...Previous year question on otosclerosis from ent based on neet pg, usmle, plab...
Previous year question on otosclerosis from ent based on neet pg, usmle, plab...Abhishek Gupta
 

More from Abhishek Gupta (8)

Medico apps resources on clinical signs
Medico apps resources on clinical signsMedico apps resources on clinical signs
Medico apps resources on clinical signs
 
Previous year question on staining based on neet pg, usmle, plab and fmge or ...
Previous year question on staining based on neet pg, usmle, plab and fmge or ...Previous year question on staining based on neet pg, usmle, plab and fmge or ...
Previous year question on staining based on neet pg, usmle, plab and fmge or ...
 
Compilation of syndromes for last minute revision tips for neet pg, usmle , p...
Compilation of syndromes for last minute revision tips for neet pg, usmle , p...Compilation of syndromes for last minute revision tips for neet pg, usmle , p...
Compilation of syndromes for last minute revision tips for neet pg, usmle , p...
 
Previous year question on pneumothorax based on neet pg, usmle, plab and fmge...
Previous year question on pneumothorax based on neet pg, usmle, plab and fmge...Previous year question on pneumothorax based on neet pg, usmle, plab and fmge...
Previous year question on pneumothorax based on neet pg, usmle, plab and fmge...
 
Previous year question on schwannoma based on neet pg, usmle, plab and fmge o...
Previous year question on schwannoma based on neet pg, usmle, plab and fmge o...Previous year question on schwannoma based on neet pg, usmle, plab and fmge o...
Previous year question on schwannoma based on neet pg, usmle, plab and fmge o...
 
Previous year question on lichen planus based on neet pg, usmle, plab and fmg...
Previous year question on lichen planus based on neet pg, usmle, plab and fmg...Previous year question on lichen planus based on neet pg, usmle, plab and fmg...
Previous year question on lichen planus based on neet pg, usmle, plab and fmg...
 
Previous year question on lichen planus based on neet pg, usmle, plab and fmg...
Previous year question on lichen planus based on neet pg, usmle, plab and fmg...Previous year question on lichen planus based on neet pg, usmle, plab and fmg...
Previous year question on lichen planus based on neet pg, usmle, plab and fmg...
 
Previous year question on otosclerosis from ent based on neet pg, usmle, plab...
Previous year question on otosclerosis from ent based on neet pg, usmle, plab...Previous year question on otosclerosis from ent based on neet pg, usmle, plab...
Previous year question on otosclerosis from ent based on neet pg, usmle, plab...
 

Recently uploaded

Call Girl Coimbatore Prisha☎️ 8250192130 Independent Escort Service Coimbatore
Call Girl Coimbatore Prisha☎️  8250192130 Independent Escort Service CoimbatoreCall Girl Coimbatore Prisha☎️  8250192130 Independent Escort Service Coimbatore
Call Girl Coimbatore Prisha☎️ 8250192130 Independent Escort Service Coimbatorenarwatsonia7
 
VIP Call Girls Pune Vani 9907093804 Short 1500 Night 6000 Best call girls Ser...
VIP Call Girls Pune Vani 9907093804 Short 1500 Night 6000 Best call girls Ser...VIP Call Girls Pune Vani 9907093804 Short 1500 Night 6000 Best call girls Ser...
VIP Call Girls Pune Vani 9907093804 Short 1500 Night 6000 Best call girls Ser...Miss joya
 
Bangalore Call Girls Marathahalli 📞 9907093804 High Profile Service 100% Safe
Bangalore Call Girls Marathahalli 📞 9907093804 High Profile Service 100% SafeBangalore Call Girls Marathahalli 📞 9907093804 High Profile Service 100% Safe
Bangalore Call Girls Marathahalli 📞 9907093804 High Profile Service 100% Safenarwatsonia7
 
Call Girls Whitefield Just Call 7001305949 Top Class Call Girl Service Available
Call Girls Whitefield Just Call 7001305949 Top Class Call Girl Service AvailableCall Girls Whitefield Just Call 7001305949 Top Class Call Girl Service Available
Call Girls Whitefield Just Call 7001305949 Top Class Call Girl Service Availablenarwatsonia7
 
Russian Call Girl Brookfield - 7001305949 Escorts Service 50% Off with Cash O...
Russian Call Girl Brookfield - 7001305949 Escorts Service 50% Off with Cash O...Russian Call Girl Brookfield - 7001305949 Escorts Service 50% Off with Cash O...
Russian Call Girl Brookfield - 7001305949 Escorts Service 50% Off with Cash O...narwatsonia7
 
Aspirin presentation slides by Dr. Rewas Ali
Aspirin presentation slides by Dr. Rewas AliAspirin presentation slides by Dr. Rewas Ali
Aspirin presentation slides by Dr. Rewas AliRewAs ALI
 
Kesar Bagh Call Girl Price 9548273370 , Lucknow Call Girls Service
Kesar Bagh Call Girl Price 9548273370 , Lucknow Call Girls ServiceKesar Bagh Call Girl Price 9548273370 , Lucknow Call Girls Service
Kesar Bagh Call Girl Price 9548273370 , Lucknow Call Girls Servicemakika9823
 
Russian Call Girls in Pune Tanvi 9907093804 Short 1500 Night 6000 Best call g...
Russian Call Girls in Pune Tanvi 9907093804 Short 1500 Night 6000 Best call g...Russian Call Girls in Pune Tanvi 9907093804 Short 1500 Night 6000 Best call g...
Russian Call Girls in Pune Tanvi 9907093804 Short 1500 Night 6000 Best call g...Miss joya
 
Call Girls Yelahanka Just Call 7001305949 Top Class Call Girl Service Available
Call Girls Yelahanka Just Call 7001305949 Top Class Call Girl Service AvailableCall Girls Yelahanka Just Call 7001305949 Top Class Call Girl Service Available
Call Girls Yelahanka Just Call 7001305949 Top Class Call Girl Service Availablenarwatsonia7
 
Low Rate Call Girls Ambattur Anika 8250192130 Independent Escort Service Amba...
Low Rate Call Girls Ambattur Anika 8250192130 Independent Escort Service Amba...Low Rate Call Girls Ambattur Anika 8250192130 Independent Escort Service Amba...
Low Rate Call Girls Ambattur Anika 8250192130 Independent Escort Service Amba...narwatsonia7
 
Call Girls In Andheri East Call 9920874524 Book Hot And Sexy Girls
Call Girls In Andheri East Call 9920874524 Book Hot And Sexy GirlsCall Girls In Andheri East Call 9920874524 Book Hot And Sexy Girls
Call Girls In Andheri East Call 9920874524 Book Hot And Sexy Girlsnehamumbai
 
Russian Call Girls Chennai Madhuri 9907093804 Independent Call Girls Service ...
Russian Call Girls Chennai Madhuri 9907093804 Independent Call Girls Service ...Russian Call Girls Chennai Madhuri 9907093804 Independent Call Girls Service ...
Russian Call Girls Chennai Madhuri 9907093804 Independent Call Girls Service ...Nehru place Escorts
 
CALL ON ➥9907093804 🔝 Call Girls Baramati ( Pune) Girls Service
CALL ON ➥9907093804 🔝 Call Girls Baramati ( Pune)  Girls ServiceCALL ON ➥9907093804 🔝 Call Girls Baramati ( Pune)  Girls Service
CALL ON ➥9907093804 🔝 Call Girls Baramati ( Pune) Girls ServiceMiss joya
 
Call Girls Service Bellary Road Just Call 7001305949 Enjoy College Girls Service
Call Girls Service Bellary Road Just Call 7001305949 Enjoy College Girls ServiceCall Girls Service Bellary Road Just Call 7001305949 Enjoy College Girls Service
Call Girls Service Bellary Road Just Call 7001305949 Enjoy College Girls Servicenarwatsonia7
 
Call Girl Chennai Indira 9907093804 Independent Call Girls Service Chennai
Call Girl Chennai Indira 9907093804 Independent Call Girls Service ChennaiCall Girl Chennai Indira 9907093804 Independent Call Girls Service Chennai
Call Girl Chennai Indira 9907093804 Independent Call Girls Service ChennaiNehru place Escorts
 
Vip Call Girls Anna Salai Chennai 👉 8250192130 ❣️💯 Top Class Girls Available
Vip Call Girls Anna Salai Chennai 👉 8250192130 ❣️💯 Top Class Girls AvailableVip Call Girls Anna Salai Chennai 👉 8250192130 ❣️💯 Top Class Girls Available
Vip Call Girls Anna Salai Chennai 👉 8250192130 ❣️💯 Top Class Girls AvailableNehru place Escorts
 
Call Girls Chennai Megha 9907093804 Independent Call Girls Service Chennai
Call Girls Chennai Megha 9907093804 Independent Call Girls Service ChennaiCall Girls Chennai Megha 9907093804 Independent Call Girls Service Chennai
Call Girls Chennai Megha 9907093804 Independent Call Girls Service ChennaiNehru place Escorts
 
Call Girls Horamavu WhatsApp Number 7001035870 Meeting With Bangalore Escorts
Call Girls Horamavu WhatsApp Number 7001035870 Meeting With Bangalore EscortsCall Girls Horamavu WhatsApp Number 7001035870 Meeting With Bangalore Escorts
Call Girls Horamavu WhatsApp Number 7001035870 Meeting With Bangalore Escortsvidya singh
 

Recently uploaded (20)

Call Girl Coimbatore Prisha☎️ 8250192130 Independent Escort Service Coimbatore
Call Girl Coimbatore Prisha☎️  8250192130 Independent Escort Service CoimbatoreCall Girl Coimbatore Prisha☎️  8250192130 Independent Escort Service Coimbatore
Call Girl Coimbatore Prisha☎️ 8250192130 Independent Escort Service Coimbatore
 
VIP Call Girls Pune Vani 9907093804 Short 1500 Night 6000 Best call girls Ser...
VIP Call Girls Pune Vani 9907093804 Short 1500 Night 6000 Best call girls Ser...VIP Call Girls Pune Vani 9907093804 Short 1500 Night 6000 Best call girls Ser...
VIP Call Girls Pune Vani 9907093804 Short 1500 Night 6000 Best call girls Ser...
 
Bangalore Call Girls Marathahalli 📞 9907093804 High Profile Service 100% Safe
Bangalore Call Girls Marathahalli 📞 9907093804 High Profile Service 100% SafeBangalore Call Girls Marathahalli 📞 9907093804 High Profile Service 100% Safe
Bangalore Call Girls Marathahalli 📞 9907093804 High Profile Service 100% Safe
 
Call Girls Whitefield Just Call 7001305949 Top Class Call Girl Service Available
Call Girls Whitefield Just Call 7001305949 Top Class Call Girl Service AvailableCall Girls Whitefield Just Call 7001305949 Top Class Call Girl Service Available
Call Girls Whitefield Just Call 7001305949 Top Class Call Girl Service Available
 
Russian Call Girl Brookfield - 7001305949 Escorts Service 50% Off with Cash O...
Russian Call Girl Brookfield - 7001305949 Escorts Service 50% Off with Cash O...Russian Call Girl Brookfield - 7001305949 Escorts Service 50% Off with Cash O...
Russian Call Girl Brookfield - 7001305949 Escorts Service 50% Off with Cash O...
 
Aspirin presentation slides by Dr. Rewas Ali
Aspirin presentation slides by Dr. Rewas AliAspirin presentation slides by Dr. Rewas Ali
Aspirin presentation slides by Dr. Rewas Ali
 
Kesar Bagh Call Girl Price 9548273370 , Lucknow Call Girls Service
Kesar Bagh Call Girl Price 9548273370 , Lucknow Call Girls ServiceKesar Bagh Call Girl Price 9548273370 , Lucknow Call Girls Service
Kesar Bagh Call Girl Price 9548273370 , Lucknow Call Girls Service
 
Russian Call Girls in Pune Tanvi 9907093804 Short 1500 Night 6000 Best call g...
Russian Call Girls in Pune Tanvi 9907093804 Short 1500 Night 6000 Best call g...Russian Call Girls in Pune Tanvi 9907093804 Short 1500 Night 6000 Best call g...
Russian Call Girls in Pune Tanvi 9907093804 Short 1500 Night 6000 Best call g...
 
Call Girls Yelahanka Just Call 7001305949 Top Class Call Girl Service Available
Call Girls Yelahanka Just Call 7001305949 Top Class Call Girl Service AvailableCall Girls Yelahanka Just Call 7001305949 Top Class Call Girl Service Available
Call Girls Yelahanka Just Call 7001305949 Top Class Call Girl Service Available
 
Low Rate Call Girls Ambattur Anika 8250192130 Independent Escort Service Amba...
Low Rate Call Girls Ambattur Anika 8250192130 Independent Escort Service Amba...Low Rate Call Girls Ambattur Anika 8250192130 Independent Escort Service Amba...
Low Rate Call Girls Ambattur Anika 8250192130 Independent Escort Service Amba...
 
Call Girls In Andheri East Call 9920874524 Book Hot And Sexy Girls
Call Girls In Andheri East Call 9920874524 Book Hot And Sexy GirlsCall Girls In Andheri East Call 9920874524 Book Hot And Sexy Girls
Call Girls In Andheri East Call 9920874524 Book Hot And Sexy Girls
 
Russian Call Girls Chennai Madhuri 9907093804 Independent Call Girls Service ...
Russian Call Girls Chennai Madhuri 9907093804 Independent Call Girls Service ...Russian Call Girls Chennai Madhuri 9907093804 Independent Call Girls Service ...
Russian Call Girls Chennai Madhuri 9907093804 Independent Call Girls Service ...
 
CALL ON ➥9907093804 🔝 Call Girls Baramati ( Pune) Girls Service
CALL ON ➥9907093804 🔝 Call Girls Baramati ( Pune)  Girls ServiceCALL ON ➥9907093804 🔝 Call Girls Baramati ( Pune)  Girls Service
CALL ON ➥9907093804 🔝 Call Girls Baramati ( Pune) Girls Service
 
Call Girls Service Bellary Road Just Call 7001305949 Enjoy College Girls Service
Call Girls Service Bellary Road Just Call 7001305949 Enjoy College Girls ServiceCall Girls Service Bellary Road Just Call 7001305949 Enjoy College Girls Service
Call Girls Service Bellary Road Just Call 7001305949 Enjoy College Girls Service
 
Call Girl Chennai Indira 9907093804 Independent Call Girls Service Chennai
Call Girl Chennai Indira 9907093804 Independent Call Girls Service ChennaiCall Girl Chennai Indira 9907093804 Independent Call Girls Service Chennai
Call Girl Chennai Indira 9907093804 Independent Call Girls Service Chennai
 
Vip Call Girls Anna Salai Chennai 👉 8250192130 ❣️💯 Top Class Girls Available
Vip Call Girls Anna Salai Chennai 👉 8250192130 ❣️💯 Top Class Girls AvailableVip Call Girls Anna Salai Chennai 👉 8250192130 ❣️💯 Top Class Girls Available
Vip Call Girls Anna Salai Chennai 👉 8250192130 ❣️💯 Top Class Girls Available
 
Russian Call Girls in Delhi Tanvi ➡️ 9711199012 💋📞 Independent Escort Service...
Russian Call Girls in Delhi Tanvi ➡️ 9711199012 💋📞 Independent Escort Service...Russian Call Girls in Delhi Tanvi ➡️ 9711199012 💋📞 Independent Escort Service...
Russian Call Girls in Delhi Tanvi ➡️ 9711199012 💋📞 Independent Escort Service...
 
Escort Service Call Girls In Sarita Vihar,, 99530°56974 Delhi NCR
Escort Service Call Girls In Sarita Vihar,, 99530°56974 Delhi NCREscort Service Call Girls In Sarita Vihar,, 99530°56974 Delhi NCR
Escort Service Call Girls In Sarita Vihar,, 99530°56974 Delhi NCR
 
Call Girls Chennai Megha 9907093804 Independent Call Girls Service Chennai
Call Girls Chennai Megha 9907093804 Independent Call Girls Service ChennaiCall Girls Chennai Megha 9907093804 Independent Call Girls Service Chennai
Call Girls Chennai Megha 9907093804 Independent Call Girls Service Chennai
 
Call Girls Horamavu WhatsApp Number 7001035870 Meeting With Bangalore Escorts
Call Girls Horamavu WhatsApp Number 7001035870 Meeting With Bangalore EscortsCall Girls Horamavu WhatsApp Number 7001035870 Meeting With Bangalore Escorts
Call Girls Horamavu WhatsApp Number 7001035870 Meeting With Bangalore Escorts
 

22-year-old patient's pre-anaesthetic review for epilepsy history

  • 1. 22-year-old Suma, with a history of epilepsy is undergoing the pre-anaesthetic review. The use of which inhalational agent is contraindicated in this patient? A: Halothane B: Enfluran e C: Sevoflurane D: Isoflurane Correct Ans:B Explanation Enflurane can precipitate generalized tonic clonic seizure in epileptics. It is however safer than halothane, causes less myocardiac depression and less hypotension. Sample Previous Year Question on Epilepsy based on previous Year Questions of NEET PG, USMLE,PLAB,FMGE (MCI Screening). Please visit www.medicoapps.org for more such Quizzes A patient presented with short lasting episodic behavioural changes which include agitation and dream like state with thrashing movements of his limbs. He does not recall these episodes and has no apparent precipitating factor. Which of the following is the most likely diagnosis? A: Panic episodes B: Schizophreni a C: Temporal lobe epilepsy D: Dissociative disorder Correct Ans:C Explanation The patient in the question has most likely suffered an attack of temporal lobe epilepsy. The loss of consciousness need not always be present but it is associated with loss of memory for the episode. In temporal lobe seizure, the patient usually experiences auras or warning signs, epigastric discomfort, olfactory hallucination, sensation of deja vu. A dream like state is often a feature of TLE. There may be loss of consciousness abnormal movement of mouth, and rarely abnormal movement of body. Frontal lobe epilepsy is also associated with episodes of agitation known as ‘intermittent explosive disorder’.
  • 2. Ref: Clinical Neuroanatomy By Stephen G. Waxman, 26th Edition, Chapter 19 Sample Previous Year Question on Epilepsy based on previous Year Questions of NEET PG, USMLE,PLAB,FMGE (MCI Screening). Please visit www.medicoapps.org for more such Quizzes Gustatory hallucinations are most commonly associated with: A: Temporal lobe epilepsy B: Grand mal epilepsy C: Anxiety disorders D: Tobacco dependence Correct Ans:A Explanation Gustatory hallucinations are most commonly associated with temporal lobe lesions, especially uncinate gyrus seizures. Patients report experiencing bitter, sweet, salty, tobacco-like, metallic or indescribable strange tastes. They are found in 4% of seizure patients with temporal lobe foci. Ref: Clinical neuropsychology 4th Ed By Kenneth M.Heilman, Page 488. Sample Previous Year Question on Epilepsy based on previous Year Questions of NEET PG, USMLE,PLAB,FMGE (MCI Screening). Please visit www.medicoapps.org for more such Quizzes A 27 year old man with epilepsy presents with complaints of persistent lethargy and occasional feelings of intoxication, although he does not consume alcohol. The patient is currently taking phenobarbital for the long-term management of tonic-clonic seizures. In addition, he was recently started on a medication for the treatment of gastroesophageal reflux disease. Which of the following agents was he most likely prescribed? A: Cimetidin e B: Famotidine C: Lansoprazole D: Ranitidine Correct Ans:A Explanation
  • 3. Cimetidine is an H2-receptor antagonist indicated for the short-term and maintenance treatment of duodenal and gastric ulceration, as well as gastroesophageal reflux disease. One of the primary disadvantages of using this agent, with respect to other H2-receptor antagonists famotidine and ranitidine, is that it is a relatively potent hepatic enzyme inhibitor. Therefore, this medication is likely to decrease the metabolism of other hepatically metabolized medications, such as phenobarbital. When the metabolism of a medication is decreased, the blood levels will increase, leading to an extension of the therapeutic effect and/or toxicity. Phenobarbital is a barbiturate indicated for the treatment of tonic-clonic seizures and status epilepticus. When the blood concentration of this medication increases, lethargy and feelings of intoxication may occur. Lansoprazole is a gastric acid proton-pump inhibitor indicated for the short-term and maintenance treatment of duodenal and gastric ulceration, as well as gastroesophageal reflux disease. This agent does not affect hepatic enzymes. Ref: McQuaid K.R. (2012). Chapter 62. Drugs Used in the Treatment of Gastrointestinal Diseases. In B.G. Katzung, S.B. Masters, A.J. Trevor (Eds),Basic & Clinical Pharmacology, 12e. Sample Previous Year Question on Epilepsy based on previous Year Questions of NEET PG, USMLE,PLAB,FMGE (MCI Screening). Please visit www.medicoapps.org for more such Quizzes At autopsy, a body is found to have copious fine leathery froth in mouth and nostrils which increased on pressure over chest. Which of the following was the most likely cause of death? A: Epilepsy B: Hangin g C: Drowning D: Opium poisoning Correct Ans:C Explanation Presence of fine, white, leathery froth seen at the mouth and nostrils is one of the most characteristic external signs of drowning. It is white or rarely blood stained, lather-like, abundant and increase in amount with compression of chest. Even after wiping it gradually reappears, especially if pressure is applied to the chest. Froth is also seen in strangulation, acute pulmonary edema, electric shock, during epileptic fit, in opium poisoning and putrefaction. But in all these cases the quantity of froth is not as large as in drowning, and the bubbles are also much smaller.
  • 4. Ref: Parikh’s Textbook of Medical Jurisprudence, Forensic Medicine and Toxicology, 6th Edition, Pages 3.66-3.67 ; Essentials of Forensic Medicine and Toxicology By Dr K S Narayan Reddy, 27th Edition, Pages 325-6 Sample Previous Year Question on Epilepsy based on previous Year Questions of NEET PG, USMLE,PLAB,FMGE (MCI Screening). Please visit www.medicoapps.org for more such Quizzes Corpus callosotomy is useful in treatment of: A: Epilepsy B: Strok e C: Alzheimer’s disease D: none of the above Correct Ans:A Explanation Patients with generalized seizures, atonic seizures associated with drop attacks, or absence seizures, who are found to have bilaterally coordinated pathologic cortical discharges on EEG and who fail AED therapy, may be candidates for corpus callosotomy. The corpus callosum is a large white matter tract that connects the cerebral hemispheres. Loss of consciousness requires simultaneous seizure activity in both hemispheres. Focal or partial seizures may spread via the corpus callosum to the contralateral hemisphere, causing generalization and loss of consciousness. Division of the corpus callosum can interrupt this spread. Ref: Schwartz’s principle of surgery 9th edition, chapter 42. Sample Previous Year Question on Epilepsy based on previous Year Questions of NEET PG, USMLE,PLAB,FMGE (MCI Screening). Please visit www.medicoapps.org for more such Quizzes Which of the following methods of contraception should be avoided in women with epilepsy? A: Oral contraceptive pills B: IUCD C: Condoms D: Diaphragm
  • 5. Correct Ans:A Explanation All the antileptics (except for sodium valproate and clonazepam) have the property to induce the enzyme complex which metabolizes the oral contraceptives. Hence it is better avoided in patients with a history of epilepsy on medications. The other three methods of contraception mentioned in the question have no such contraindication. Ref: Dutta textbook of Obstetrics, 6th Edition, Page 545; Textbook of Gynaecology By Rao, Page 183-184 Sample Previous Year Question on Epilepsy based on previous Year Questions of NEET PG, USMLE,PLAB,FMGE (MCI Screening). Please visit www.medicoapps.org for more such Quizzes Which of the following drugs is not used in Juvenile Myoclonic Epilepsy (JME)? A: Topiramat e B: Zonisamide C: Carbamezapine D: Valproate Correct Ans:C Explanation Carbamezipine, Phenytoin, Oxcarbazepine are few drugs which are capable of aggravating Juvenile Myoclonic Epilepsy. Ref: Epileptic Syndromes in Infancy, Childhood and Adolescence By Joseph Roger, Michelle Bureau, Charlotte Dravet, Pierre Genton, Page 382 Sample Previous Year Question on Epilepsy based on previous Year Questions of NEET PG, USMLE,PLAB,FMGE (MCI Screening). Please visit www.medicoapps.org for more such Quizzes
  • 6. All of the following factors are associated with a substantially greater risk of developing epilepsy after febrile seizures, except: A: Complex Febrile seizures B: Early age of onset C: Developmental abnormalities D: Positive family History of Epilepsy Correct Ans:B Explanation Factors increasing risk of epilepsy after febrile seizures include family history of epilepsy, a typical seizures/complex nature of seizures and presence of a neurodevelopmental abnormality. Early age on onset increases the risk of recurrence of febrile convulsions. Ref: Febrile Seizures By Tallie Z. Baram, Shlomo Shinnar, Page 63; Essential of Pediatrics By O P Ghai, 6th Edition, Page 509 Sample Previous Year Question on Epilepsy based on previous Year Questions of NEET PG, USMLE,PLAB,FMGE (MCI Screening). Please visit www.medicoapps.org for more such Quizzes An adolescent is brought to the emergency department following an episode of myoclonic jerks at morning after waking up. His consciousness was not impaired. His EEG shows generalized 3-4 Hz spike and slow wave complexes. Most probable diagnosis is? A: Generalized tonic clonic seizure B: Absent seizure C: Temporal lobe epilepsy D: Juvenile myoclonic epilepsy Correct Ans:D Explanation Juvenile myoclonic epilepsy is a subtype of idiopathic generalized epilepsy with onset usually between 8 and 20 years of age. Myoclonic jerks, especially in the morning, are of variable intensity ranging from simple twitching (“flying saucer syndrome”) to falls; consciousness is not impaired in it. It is precipitated by alcohol and sleep deprivation. Patients will have normal intelligence. The typical interictal EEG abnormality consists of a generalized 4- to 6-Hz spike or polyspike and slow-wave discharges lasting 1-20 seconds. Usually, 1-3 spikes precede each slow wave. Also know:
  • 7.  In Generalized tonic clonic seizure EEG shows a normal background with generalized epileptiform discharges such as spike or polyspike wave complexes at 2.5 to 4 Hz.  During absence seizures there is an abrupt onset of bilaterally synchronous and symmetrical 3 Hz spike-wave discharge, irrespective of whether typical absences are simple or complex.  Ictal recordings from patients with typical temporal lobe epilepsy usually exhibit 5-7 Hz, rhythmic, sharp theta activity, maximal in the sphenoidal and the basal temporal electrodes on the side of seizure origin. Ref: A-Z of Neurological Practice: A Guide to Clinical Neurology By Andrew J. Larner, Alasdair J Coles, Neil J. Scolding, Roger A Barker, 2011, Page 368 ; Clinical Electroencephalography by Misra,2005, Page 188 Sample Previous Year Question on Epilepsy based on previous Year Questions of NEET PG, USMLE,PLAB,FMGE (MCI Screening). Please visit www.medicoapps.org for more such Quizzes Which one of the following is the characteristic feature of juvenile myoclonic epilepsy? A: Myoclonic seizures frequently occur in morning B: Complete remission is common C: Response to anticonvulsants is poor D: Associated absence seizures are present in majority of patients Correct Ans:A Explanation Juvenile myoclonic epilepsy (JME) is a generalized seizure disorder of unknown cause that appears in early adolescence and is usually characterized by bilateral myoclonic jerks that may be single or repetitive. The myoclonic seizures are most frequent in the morning after awakening and can be provoked by sleep deprivation. Consciousness is preserved unless the myoclonus is especially severe. Many patients also experience generalized tonic-clonic seizures, and up to one-third have absence seizures. Although complete remission is relatively uncommon, the seizures respond well to appropriate anticonvulsant medication. There is often a family history of epilepsy, and genetic linkage studies suggest a polygenic cause.
  • 8. Ref: Lowenstein D.H. (2012). Chapter 369. Seizures and Epilepsy. In D.L. Longo, A.S. Fauci, D.L. Kasper, S.L. Hauser, J.L. Jameson, J. Loscalzo (Eds), Harrison's Principles of Internal Medicine, 18e. Sample Previous Year Question on Epilepsy based on previous Year Questions of NEET PG, USMLE,PLAB,FMGE (MCI Screening). Please visit www.medicoapps.org for more such Quizzes Which of the following statements is incorrect in relation to pregnant women with epilepsy? A: The rate of congenital malformation is increased in the offspring of women with epilepsy B: Seizure frequency increases in approximately 70% of women C: Breast feeding is safe with most anticonvulsants D: Folic acid supplementation may reduce the risk of neural tube defect Correct Ans:B Explanation Frequency of convulsions is unchanged in majority. Ref:Textbook of Gynecology By D C Dutta, 6th Edition, Page 298; Harrison’s Principles of Internal Medicine, 16th Edition, Page 2371 Sample Previous Year Question on Epilepsy based on previous Year Questions of NEET PG, USMLE,PLAB,FMGE (MCI Screening). Please visit www.medicoapps.org for more such Quizzes Vitamin deficient is most commonly seen in a pregnant mother who is on phenytoin therapy for epilepsy? A: Vitamin B6 B: Vitamin B12 C: Vitamin A D: Folic acid Correct Ans:D Explanation
  • 9. Phenytoin interferes with metabolism of folate and patients taking phenytoin may become deficient in folic acid. Phenytoin decreases absorption of folic acid and increases its excretion. Ref: Advanced Therapy in Epilepsy By Wheless, James W. Wheless, James Willmore, Roger A. Brumback, 2009, Page 259. Sample Previous Year Question on Epilepsy based on previous Year Questions of NEET PG, USMLE,PLAB,FMGE (MCI Screening). Please visit www.medicoapps.org for more such Quizzes Triad of Tuberous Sclerosis includes all, except: A: Epilepsy B: Low intelligence C: Hydrocephalus D: Adenoma sebaceum Correct Ans:C Explanation Diagnostic triad of Tuberous sclerosis includes epilepsy, mental retardation and adenoma sebaceum (facial angiofibroma). Tuberous sclerosis (TS) is an autosomal dominant disorder which result from mutations in either the TSC1 gene encoding hamartin or the TSC2 gene encoding tuberin. Hamartin and tubulin form a complex which negatively regulate cell growth and proliferation through inhibition of mTOR. Ref: Harrison’s Internal Medicine, 18th Edition, Chapter 284; The 5-Minute Neurology Consult By D. Joanne Lynn, Page 430; Tuberous Sclerosis Complex: Genes, Clinical Features and Therapeutics By David J. Kwiatkowsk, Page 221 Sample Previous Year Question on Epilepsy based on previous Year Questions of NEET PG, USMLE,PLAB,FMGE (MCI Screening). Please visit www.medicoapps.org for more such Quizzes In pediatric patients, the risk of developing post-traumatic epilepsy is significantly increased by:
  • 10. A: Brief loss of consciousness B: Acute intracranial hemorrhage C: Retrograde amnesia D: Post-traumatic vomiting Correct Ans:B Explanation Late post-traumatic epilepsy is diagnosed when a seizure occurs for the first time more than one week after a head injury. Factors that correlate with an increased risk of developing post-traumatic epilepsy include presence of a depressed skull fracture, acute intracranial hemorrhage, cerebral contusion, or unconsciousness lasting more than 24 hours. Because the risk of a subsequent seizure is approximately 75%, acute and chronic treatment with anticonvulsants is indicated. Loss of consciousness, retrograde amnesia, and vomiting are relatively common immediate consequences of head trauma. They are usually transient and are not highly correlated with a risk of subsequent post-traumatic seizures. Ref: Smith W.S., English J.D., Johnston S.C. (2012). Chapter 370. Cerebrovascular Diseases. In D.L. Longo, A.S. Fauci, D.L. Kasper, S.L. Hauser, J.L. Jameson, J. Loscalzo (Eds),Harrison's Principles of Internal Medicine, 18e. Sample Previous Year Question on Epilepsy based on previous Year Questions of NEET PG, USMLE,PLAB,FMGE (MCI Screening). Please visit www.medicoapps.org for more such Quizzes All of the following structures pass through optic foramen, EXCEPT: A: Optic nerve B: Ophthalmic artery C: Ophthalmic nerve D: Dural mater Correct Ans:C Explanation The optic foramen lies at the orbital apex and transmits the optic nerve and ophthalmic artery. Superior orbital fissure transmits the ophthalmic nerve. Optic foramen: Is a oval opening at the apex of the orbit. It lies between the two roots of the lesser wings of the sphenoid bone. It transmits
  • 11.  Optic nerve  Ophthalmic branch of internal carotid artery  Branches of sympathetic carotid plexus  Prolongation of optic nerve sheaths Superior orbital fissure: The superior orbital fissure is bounded by the lesser and greater wings of the sphenoid. Contents:  Superior ophthalmic vein  Oculomotor nerve (III)  Trochlear nerve (IV)  Lacrimal, frontal and nasociliary branches of Ophthalmic nerve (V1)  Abducent nerve (VI)  Orbital branch of middle meningeal artery  Recurrent branch of lacrimal artery  Superior orbital vein  Superior ophthalmic vein Inferior orbital fissure: The greater wing of the sphenoid, the maxilla, and the palatine bones of the orbit form the boundaries of the inferior orbital fissure. The fissure transmits the  Maxillary nerve and its zygomatic branch,  Infraorbital vessels,  Ascending branches from the sphenopalatine ganglion and a vein which connects the inferior ophthalmic vein with the pterygoid venous plexus. Ref: Comprehensive Manual of Ophthalmology, By Ahmed E, Page 4 Sample Previous Year Question on Foramen Part II based on previous Year Questions of NEET PG, USMLE,PLAB,FMGE (MCI Screening). Please visit www.medicoapps.org for more such Quizzes Which of the following opening in the base of the skull transmits the third branch of trigeminal nerve? A: Foramen ovale B: Foramen lacerum C: Foramen magnum D: Foramen spinosum Correct Ans:A Explanation Foramen ovale is an opening at the base of the lateral pterygoid plate. It transmits the third branch of the trigeminal nerve, the accessory meningeal artery, and occasionally the superficial petrosal nerve.
  • 12. Foramen lacerum transmits the internal carotid artery. Foramen magnum transmits the medulla and its membranes, the spinal accessory nerves, the vertebral arteries, and the anterior and posterior spinal arteries. Ref: Waxman S.G. (2010). Chapter 11. Ventricles and Coverings of the Brain. In S.G. Waxman (Ed), Clinical Neuroanatomy, 26e. Sample Previous Year Question on Foramen Part II based on previous Year Questions of NEET PG, USMLE,PLAB,FMGE (MCI Screening). Please visit www.medicoapps.org for more such Quizzes Which of the following cranial nerve travels through the jugular foramen in the base of the skull? A: 3rd branch of trigeminal nerve B: Abducens nerve C: Facial nerve D: Glossopharyngeal nerve Correct Ans:D Explanation Glossopharyngeal nerve travel through the middle compartment of the jugular foramen. Jugular foramen is formed by the petrous portion of the temporal and occipital bones. It is divided into 3 compartments. The intermediate compartment contains the glossopharyngeal, vagus, and spinal accessory nerves. The anterior compartment contains the inferior petrosal sinus and the posterior compartment contains the sigmoid sinus and meningeal branches from the occipital and ascending pharyngeal arteries. Ref: Waxman S.G. (2010). Chapter 11. Ventricles and Coverings of the Brain. In S.G. Waxman (Ed), Clinical Neuroanatomy, 26e. Sample Previous Year Question on Foramen Part II based on previous Year Questions of NEET PG, USMLE,PLAB,FMGE (MCI Screening). Please visit www.medicoapps.org for more such Quizzes Middle meningeal artery courses along which of the following opening in the base of the skull? A: Foramen ovale B: Foramen lacerum C: Foramen spinosum
  • 13. D: Foramen rotundum Correct Ans:C Explanation Middle meningeal artery and vein travels through foramen spinosum. Structures passing through openings in the skull base are: Foramen Structures Cribriform plate of ethmoid Olfactory nerves Optic foramen Optic nerve, ophthalmic artery, meninges Superior orbital fissure Oculomotor, trochlear, and abducens nerves; ophthalmic division of trigemina Foramen rotundum Maxillary division of trigeminal nerve, small artery and vein Foramen ovale Mandibular division of trigeminal nerve, vein Foramen lacerum Internal carotid artery, sympathetic plexus Foramen spinosum Middle meningeal artery and vein Internal acoustic meatus Facial and vestibulocochlear nerves, internal auditory artery Jugular foramen Glossopharyngeal, vagus, and spinal accessory nerves; sigmoid sinus Hypoglossal canal Hypoglossal nerve Foramen magnum Medulla and meninges, spinal accessory nerve, vertebral arteries, anterior an Ref: Waxman S.G. (2010). Chapter 11. Ventricles and Coverings of the Brain. In S.G. Waxman (Ed), Clinical Neuroanatomy, 26e. Sample Previous Year Question on Foramen Part II based on previous Year Questions of NEET PG, USMLE,PLAB,FMGE (MCI Screening). Please visit www.medicoapps.org for more such Quizzes Which of the following structure is passing through foramen rotundum? A: Maxillary artery B: Maxillary nerve
  • 14. C: Middle meningeal artery D: Spinal accessory nerve Correct Ans:B Explanation Foramen rotundum located posterior to the medial end of the superior orbital fissure. The foramen rotundum transmits the maxillary nerve (CN V-2) en route to the pterygopalatine fossa. CN V-2 supplies the skin, teeth, and mucosa associated with the maxillary bone. Ref: Morton D.A., Foreman K.B., Albertine K.H. (2011). Chapter 15. Scalp, Skull, and Meninges. In D.A. Morton, K.B. Foreman, K.H. Albertine (Eds), The Big Picture: Gross Anatomy. Sample Previous Year Question on Foramen Part II based on previous Year Questions of NEET PG, USMLE,PLAB,FMGE (MCI Screening). Please visit www.medicoapps.org for more such Quizzes Herniation through Foramen of Morgagni is seen in children. Foramen of Morgagni refers to an opening in: A: The brain B: The lesser omentum C: The skull D: The diaphragm Correct Ans:D Explanation The foramen of Morgagni occurs at the junction of the septum transversum and the anterior thoracic wall. This anterior, central diaphragmatic defect accounts for only 2% of diaphragmatic hernias. It may be parasternal, retrosternal, or bilateral. The defect is closed by suturing the posterior rim of diaphragm to the posterior rectus sheath, since there is no anterior diaphragm. Ref: Albanese C.T., Sylvester K.G. (2010). Chapter 43. Pediatric Surgery. In G.M. Doherty (Ed), CURRENT Diagnosis & Treatment: Surgery, 13e. Sample Previous Year Question on Foramen Part II based on previous Year Questions of NEET PG, USMLE,PLAB,FMGE (MCI Screening). Please visit www.medicoapps.org for more such Quizzes
  • 15. Which of the following circulatory changes normally take place in the newborn within 5 minutes after birth? A: Closure of the ductus arteriosus B: Closure of the foramen ovale C: Increase in pulmonary artery resistance D: All of the above Correct Ans:B Explanation With the first breath of life, pulmonary arterial resistance drops dramatically. This is due to the oxygenation of the lungs causing vasodilatation of the pulmonary vessels. Clamping of the umbilical cord doubles peripheral resistance and causes an increase in arterial blood pressure. As soon as right atrial, pressure drops below left atrial pressure, the foramen ovale will close (valve-like mechanism), establishing the adult-type blood circulation. The rising systemic and falling pulmonary artery pressure causes a flow reversal through the ductus arteriosus from right to left to left to right within minutes of birth. Complete closure of the ductus arteriosus appears to be due to a decline of local prostacyclin levels and usually occurs within 24 to 48 hours after birth. Ref: Morton D.A., Foreman K.B., Albertine K.H. (2011). Chapter 4. Heart. In D.A. Morton, K.B. Foreman, K.H. Albertine (Eds), The Big Picture: Gross Anatomy. Sample Previous Year Question on Foramen Part II based on previous Year Questions of NEET PG, USMLE,PLAB,FMGE (MCI Screening). Please visit www.medicoapps.org for more such Quizzes Foramen of Morgagni refers to an opening in: A: The brain B: The lesser omentum C: The skull D: The diaphragm Correct Ans:D Explanation Foramen of morgagni is a small triangular diaphragmatic cleft bounded medially by muscle fibers originating from the sternum and laterally by the seventh costal cartilages. It occurs due to lack of fusion or muscularization of the pleuroperitoneal membrane anteriorly leading to a defect in the costosternal trigones.
  • 16. Diaphragmatic hernia can occur anteriorly at the foramen of morgagni and posterolaterally through the Bochdalek foramen or at the esophageal hiatus. Bochdalek foramen refers to the gap between the lumbar and costal elements. Openings in the diaphragm:  The caval foramen is located on the right at the level of T8 within the central tendon.  Esophageal foramen lies at T10 level.  The aortic opening lies behind the diaphragm at its lowest point opposite to the T12 vertebrae. Ref: Teaching Atlas of Chest Imaging By Mark Parker page 757, Upper Gastrointestinal Surgery edited by John W. L. Fielding, page 118. Sample Previous Year Question on Foramen Part II based on previous Year Questions of NEET PG, USMLE,PLAB,FMGE (MCI Screening). Please visit www.medicoapps.org for more such Quizzes All of the following statements are true regarding the pudendal nerve, except: A: It is both sensory and motor B: It is derived from S2,3,4 C: It leaves the pelvis through the lesser sciatic foramen D: It leaves through lesser sciatic foramen and enter pudendal canal Correct Ans:C Explanation Pudendal nerve leaves the pelvis, to enter the gluteal region, by passing through the lower part of the greater sciatic foramen. Pudendal nerve leaves the gluteal region by passing through the lesser sciatic foramen and enters the pudendal canal, and by means of its branches supplies the external anal sphincter and muscles and skin of the perineum. Ref: Clinical anatomy for Medical Students By Richard S Snell, 6th Edition, Pages 362-3 ; B.D.Chaurasia’s Human Anatomy, 5th Edition, Volume 2, Page 365 Sample Previous Year Question on Foramen Part II based on previous Year Questions of NEET PG, USMLE,PLAB,FMGE (MCI Screening). Please visit www.medicoapps.org for more such Quizzes
  • 17. The boundaries of the interconnection between greater sac and lesser sac of peritoneum known as 'Foramen of Winslow' are all, EXCEPT: A: Caudate lobe of liver B: Inferior vena cava C: Free border of lesser omentum D: 4th part of Duodenum Correct Ans:D Explanation Interconnection between greater sac and lesser sac of peritoneum is known as Foramen of Winslow. It has the following boundaries: Superior boundary: Caudate lobe of liver Anterior boundary: Free edge of lesser omentum containing common bile duct, hepatic artery andportal vein. Inferior boundary: First part of duodenum Posterior boundary: Inferior vena cava and abdominal aorta Sample Previous Year Question on Foramen Part II based on previous Year Questions of NEET PG, USMLE,PLAB,FMGE (MCI Screening). Please visit www.medicoapps.org for more such Quizzes Structures passing through lesser sciatic foramen are all, except: A: Pudendal nerve B: Internal pudendal artery C: Nerve to obturator externus D: Tendon of obturator internus Correct Ans:C Explanation Structures passing through lesser sciatic foramen are tendon of obturator internus muscle, nerve to obturator internus, pudendal nerve, internal pudendal artery and vein. Structures passing through greater sciatic foramen are:  Piriformis  Sciatic nerve  Posterior cutaneous nerve of thigh
  • 18.  Superior and inferior gluteal nerve  Nerve to obturator internus and quadratus femoris  Pudendal nerve  Superior and inferior gluteal nerves and arteries  Internal pudendal artery and vein Sample Previous Year Question on Foramen Part II based on previous Year Questions of NEET PG, USMLE,PLAB,FMGE (MCI Screening). Please visit www.medicoapps.org for more such Quizzes Which of the following structures form the superior border of Epiploic foramen ? A: Quadrate lobe of liver B: Caudate process of caudate lobe of liver C: Porta hepatis D: First part of the duodenum Correct Ans:B Explanation In human anatomy, the omental foramen (Epiploic foramen, foramen of Winslow, or uncommonly aditus) is the passage of communication or foramen, between the greater sac (general cavity of the abdomen) and the lesser sac. Borders of Epiploic foramen: It has the following boundaries:  Anteriorly : free border of lesser omentum, bile duct, hepatic artery and portal vein.  Posteriorly : Inferior vena cava, right suprarenal gland and T12 vertebrae  Superiorly : Caudate process of caudate lobe of liver  Inferiorly : First part of the duodenum and horizontal part of hepatic artery Sample Previous Year Question on Foramen Part II based on previous Year Questions of NEET PG, USMLE,PLAB,FMGE (MCI Screening). Please visit www.medicoapps.org for more such Quizzes Which of the following structure forms the superior border of Epiploic foramen?
  • 19. A: Head of pancreas B: Caudate lobe of liver C: Lesser omentum D: IVC Correct Ans:B Explanation Caudate process of caudate lobe of the liver forms the superior border of epiploic foramen. The right margin of the lesser sac opens into the greater sac through the opening of the lesser sac called Epiploic foramen. Boundaries of epiploic foramen are formed by: Anteriorly: Free border of the lesser omentum, bile duct, hepatic artery and portal vein. Posteriorly: Inferior vena cava Superiorly: Caudate process of caudate lobe of the liver Inferiorly: First part of the duodenum. Ref: Clinical Anatomy by Regions By Richard S. Snell, 8th edn, page 208 Sample Previous Year Question on Foramen Part II based on previous Year Questions of NEET PG, USMLE,PLAB,FMGE (MCI Screening). Please visit www.medicoapps.org for more such Quizzes All of the following structures forms the boundries of epiploic foramen, EXCEPT: A: Caudate process of liver B: Inferior vena cava C: 4th part duodenum D: Free border of lesser omentum Correct Ans:C Explanation Epiploic foramen also known as foramen of winslow is a vertical slit like opening, through which the lesser sac communicates with the greater sac. Boundaries of the epiploic formen:
  • 20.  Anteriorly: by lesser omentum containing portal vein, hepatic artery and bile duct.  Posteriorly: suprarenal gland inferior vena cava and T12 vertebra.  Superiorly: caudate lobe of liver.  Inferiorly: 1st part of duodenum and hepatic artery. Ref: Cunningham's textbook of anatomy, By Daniel John Cunningham, Page 1239 Sample Previous Year Question on Foramen Part II based on previous Year Questions of NEET PG, USMLE,PLAB,FMGE (MCI Screening). Please visit www.medicoapps.org for more such Quizzes Structure that DOES NOT pass through lesser sciatic foramen: A: Pudendal nerve B: Internal pudendal vessels C: Tendon to obturator internus D: Tendon to obturator externus Correct Ans:D Explanation Lesser sciatic foramen is formed by the lesser sciatic notch and the sacrotuberous and sacrospinous ligaments. Structures that pass through the foramen are:  Tendon to obturatur internus  Nerve to obturatur internus  Pudendal nerve and  Internal pudendal artery and vein Ref: Snell's, Clinical Anatomy, 7th Edition, Page 602. Sample Previous Year Question on Foramen Part II based on previous Year Questions of NEET PG, USMLE,PLAB,FMGE (MCI Screening). Please visit www.medicoapps.org for more such Quizzes A 47 year old woman presents to the emergency department with cramping/colicky abdominal pain. The current episode of pain began several hours ago, following a fatty meal. The pain began slowly, and rose in intensity to a plateau over the course of several hours. The patient reports that she had had several other episodes of similar pain during
  • 21. the past several months, with long intervening periods of freedom from pain. On physical examination, she is noted to have tenderness to deep palpation in the right upper quadrant of the abdomen near the rib cage. The patient also reports that she is experiencing shoulder/back pain at a site she identifies near the right lower scapula, but no tenderness can be elicited during the back and shoulder examination. Following appropriate diagnostic studies, the patient is taken to the surgical suite. During the surgery, the surgeon inserts his fingers from right to left behind the hepatoduodenal ligament. As he does so, his fingers enter which of the following? A: Ampulla of Vater B: Common bile duct C: Epiploic foramen D: Greater peritoneal sac Correct Ans:C Explanation The space behind the stomach, hepatoduodenal ligament, and hepatogastric ligament is the omental bursa. This space can be entered by passing through the epiploic foramen of Winslow, as described in the question stem. Good to know:  The common bile duct enters the duodenum through the ampulla of Vater.  The hepatoduodenal ligament contains the common bile duct, the portal vein and the hepatic artery.  The greater peritoneal sac lies anterior to the stomach and hepatoduodenal ligament. Ref: Morton D.A., Foreman K.B., Albertine K.H. (2011). Chapter 8. Serous Membranes of the Abdominal Cavity. In D.A. Morton, K.B. Foreman, K.H. Albertine (Eds), The Big Picture: Gross Anatomy. Sample Previous Year Question on Foramen Part II based on previous Year Questions of NEET PG, USMLE,PLAB,FMGE (MCI Screening). Please visit www.medicoapps.org for more such Quizzes Epiploic foramen provides communication between greater and lesser sacs. The length of the epiploic foramen is: A: 5 cm B: 6 cm
  • 22. C: 4 cm D: 3 cm Correct Ans:D Explanation The greater and lesser sacs communicate with each other through the epiploic foramen (of Winslow). The foramen is about 3 cm in size and situated opposite the 12th thoracic vertebra. Ref: Morton D.A., Foreman K.B., Albertine K.H. (2011). Chapter 8. Serous Membranes of the Abdominal Cavity. In D.A. Morton, K.B. Foreman, K.H. Albertine (Eds), The Big Picture: Gross Anatomy. Sample Previous Year Question on Foramen Part II based on previous Year Questions of NEET PG, USMLE,PLAB,FMGE (MCI Screening). Please visit www.medicoapps.org for more such Quizzes Where is the ‘Foramen of Winslow’ located anatomically? A: Between greater and lesser sac B: At hilum of liver C: Transverse cervical ligament of uterus D: Pouch of Douglas Correct Ans:A Explanation The lesser omentum attaches between the liver, stomach, and proximal portion of the duodenum. It forms a sac known as the omental bursa, which forms a subdivision of the peritoneal cavity known as the lesser sac. The greater sac is the remaining part of the peritoneal cavity. The greater and lesser sacs communicate with each other through the epiploic foramen (of Winslow). Ref: Morton D.A., Foreman K.B., Albertine K.H. (2011). Chapter 8. Serous Membranes of the Abdominal Cavity. In D.A. Morton, K.B. Foreman, K.H. Albertine (Eds), The Big Picture: Gross Anatomy. Sample Previous Year Question on Foramen Part II based on previous Year Questions of NEET PG, USMLE,PLAB,FMGE (MCI Screening). Please visit www.medicoapps.org for more such Quizzes
  • 23. Which of the following is the anatomical location of foramen of winslow? A: Between greater and lesser sac B: At hilum of liver C: Transverse cervical ligament of uterus D: Pouch of Douglas Correct Ans:A Explanation The lesser omentum forms a sac known as the omental bursa, which forms a subdivision of the peritoneal cavity known as the lesser sac. The greater sac is the remaining part of the peritoneal cavity. The greater and lesser sacs communicate with each other through the epiploic foramen (of Winslow). Also know: The omentum refers to modified mesenteries associated with the stomach and liver.  Greater omentum. An apron-like fold of mesentery that attaches between the transverse colon to the greater curvature of the stomach.  Lesser omentum. Mesentery that attaches between the liver, stomach, and proximal portion of the duodenum. As a result, the lesser omentum is also referred to as thehepatogastric ligament and hepatoduodenal ligament. Ref: Morton D.A., Foreman K.B., Albertine K.H. (2011). Chapter 8. Serous Membranes of the Abdominal Cavity. In D.A. Morton, K.B. Foreman, K.H. Albertine (Eds), The Big Picture: Gross Anatomy. Sample Previous Year Question on Foramen Part II based on previous Year Questions of NEET PG, USMLE,PLAB,FMGE (MCI Screening). Please visit www.medicoapps.org for more such Quizzes Where is the anatomical location of Foramen of Winslow? A: Between greater and lesser sac B: At hilum of liver C: Transverse cervical ligament of uterus D: Pouch of Douglas Correct Ans:A Explanation
  • 24. The lesser omentum forms a sac known as the omental bursa, which forms a subdivision of the peritoneal cavity known as the lesser sac. The greater sac is the remaining part of the peritoneal cavity. The greater and lesser sacs communicate with each other through the epiploic foramen (of Winslow). Boundaries of epiploic foramen:  Anteriorly: The free border of the lesser omentum, containing the bile duct, the hepatic artery, and the portal vein  Posteriorly: Inferior vena cava  Superiorly: The caudate process of the caudate lobe of the liver  Inferiorly: The first part of the duodenum Ref: Morton D.A., Foreman K.B., Albertine K.H. (2011). Chapter 8. Serous Membranes of the Abdominal Cavity. In D.A. Morton, K.B. Foreman, K.H. Albertine (Eds), The Big Picture: Gross Anatomy. Sample Previous Year Question on Foramen Part II based on previous Year Questions of NEET PG, USMLE,PLAB,FMGE (MCI Screening). Please visit www.medicoapps.org for more such Quizzes The posterior relation of epiploic foramen is: A: Hepatic artery B: Inferior vena cava C: Common bile duct D: Portal vein Correct Ans:B Explanation Epiploic foramen or aditus to lesser sac communicates lesser sac to greater sac. It is bounded posteriorly by inferior vena cava, suprarenal gland and T12 vertebra. Epiploic foramen: also known as foramen of winslow is a vertical slit like opening, through which the lesser sac communicates with the greater sac. It is situated at the upper wall of lesser sac at the level of T12. Boundaries of the epiploic foramen:  Anteriorly: right free margin of lesser omentum containing portal vein, hepatic artery and bile duct
  • 25.  Posteriorly: suprarenal gland, inferior vena cava and T12 vertebra  Superiorly: caudate lobe of liver  Inferiorly: 1st part of duodenum and hepatic artery Ref: Cunningham's textbook of anatomy, By Daniel John Cunningham, Page 1239 Sample Previous Year Question on Foramen Part III based on previous Year Questions of NEET PG, USMLE,PLAB,FMGE (MCI Screening). Please visit www.medicoapps.org for more such Quizzes All the following structures form boundaries of Epiploic foramen, EXCEPT: A: Portal vein B: Inferior vena cava C: Quadrate lobe of liver D: First part of duodenum Correct Ans:C Explanation The right margin of the lesser sac opens into the greater sac through the epiploic foramen. The superior boundary of the foramen is foramen by caudate lobe of the liver. Structures forming boundaries of the epiploic foramen are: Anteriorly: free border of lesser omentum, bile duct, hepatic artery and portal vein. Posteriorly: Inferior vena cava Superiorly: Caudate process of caudate lobe of liver Inferiorly: First part of duodenum Ref: Clinical Anatomy by Regions By Richard S. Snell page 208. Sample Previous Year Question on Foramen Part III based on previous Year Questions of NEET PG, USMLE,PLAB,FMGE (MCI Screening). Please visit www.medicoapps.org for more such Quizzes All of the following structures pass through the greater sciatic foramen, EXCEPT: A: Piriformis muscle B: Pudendal nerve C: Inferior gluteal vessel D: Internal pudendal vessel
  • 26. Correct Ans:B Explanation Pudendal nerve courses through the lesser sciatic foramen. Structures passing through the lesser sciatic foramen are:  Pudendal nerve  Internal pudendal vessels  Obturator internus tendon Structures passing through the greater sciatic foramen are:  Sciatic nerve  Piriformis muscle  Internal pudendal and inferior gluteal vessels  Other branches of the sacral nerve plexus Ref: Hoffman B.L., Schorge J.O., Schaffer J.I., Halvorson L.M., Bradshaw K.D., Cunningham F.G., Calver L.E. (2012). Chapter 38. Anatomy. Sample Previous Year Question on Foramen Part III based on previous Year Questions of NEET PG, USMLE,PLAB,FMGE (MCI Screening). Please visit www.medicoapps.org for more such Quizzes Which of the following is TRUE about ‘Foramen of Winslow’? A: It is seen between greater and lesser sac B: Anteriorly bound by 2nd part of duodenum C: Seen in transverse cervical ligament of uterus D: Posteriorly bound by pouch of Douglas Correct Ans:A Explanation The epiploic foramen of Winslow or the aditus to the lesser sac is the only communication between greater and lesser sac. It is bounded,  Anteriorly by hepatoduodenal ligament  Posteriorly by inferior venacava  Superiorly by caudate lobe of liver  Inferiorly by first part of duodenum Ref: Last's Anatomy: Regional and Applied By Chummy S. Sinnatamby, 2011, Chapter 5. Sample Previous Year Question on Foramen Part III based on previous Year Questions of NEET PG, USMLE,PLAB,FMGE (MCI Screening). Please visit www.medicoapps.org for more such Quizzes
  • 27. All of the following passing through lesser sciatic foramen, except: A: Pudendal nerve B: Internal pudendal vessels C: Nerve to obturator internus D: Inferior gluteal vessels Correct Ans:D Explanation The lesser sciatic foramen is formed by the lesser sciatic notch of the pelvic bone, the ischial,the sacrospinous ligament and the sacrotuberous ligament. It transmits the following structures:  The tendon of the Obturator internus  Internal pudendal vessels  Pudendal nerve  Nerve to the obturator internus Ref: Morton D.A., Foreman K.B., Albertine K.H. (2011). Chapter 12. Pelvis and Perineum. In D.A. Morton, K.B. Foreman, K.H. Albertine (Eds), The Big Picture: Gross Anatomy. Sample Previous Year Question on Foramen Part III based on previous Year Questions of NEET PG, USMLE,PLAB,FMGE (MCI Screening). Please visit www.medicoapps.org for more such Quizzes All of the following structures passes through lesser sciatic foramen, EXCEPT: A: Inferior gluteal vessels B: Internal pudendal vessels C: Pudendal nerve D: Nerve to obturator internus Correct Ans:A Explanation Structures passing through the lesser sciatic foramen are tendon of obturator internus, internal pudendal vessels and pudendal nerve.
  • 28. Structures passing through greater sciatic foramen are:  Piriformis muscle  Superior and inferior gluteal vessel  Internal pudendal vessel  Pudendal nerve  Sciatic nerve  Posterior femoral cutaneous nerve  Nerve to obturator internus  Quadratus femoris Structures passing through both greater and lesser sciatic foramen:  Pudendal nerve  Internal pudendal vessels  Nerve to obturator internus Ref: BRS Gross Anatomy By Kyung Won Chung, page 97 Sample Previous Year Question on Foramen Part III based on previous Year Questions of NEET PG, USMLE,PLAB,FMGE (MCI Screening). Please visit www.medicoapps.org for more such Quizzes The middle meningeal artery enters the cranium through the: A: Foramen spinosum B: Foramen rotundum C: Foramen magnum D: Foramen ovale Correct Ans:A Explanation The middle meningeal artery is the largest of the arteries supplying the dura mater. It is a branch of the maxillary artery and enters the cranium through the foramen spinosum. Ref: Gray's Basic Anatomy International Ed, page 429 Sample Previous Year Question on Foramen Part III based on previous Year Questions of NEET PG, USMLE,PLAB,FMGE (MCI Screening). Please visit www.medicoapps.org for more such Quizzes
  • 29. Which of the following foramen is responsible for the drainage of CSF from lateral to third ventricle? A: Foramen of Monro B: Foramen of Luschka C: Foramen of Magendie D: Cerebral aqueduct Correct Ans:A Explanation The two interventricular foramens, or foramens of Monro, are apertures between the column of the fornix and the anterior end of the thalamus. The two lateral ventricles communicate with the third ventricle through these foramens and drainage of CSF occurs. The lateral aperture (foramen of Luschka) is the opening of the lateral recess into the subarachnoid space near the flocculus of the cerebellum. A tuft of choroid plexus is commonly present in the aperture and partly obstructs the flow of CSF from the fourth ventricle to the subarachnoid space. The medial aperture (foramen of Magendie) is an opening in the caudal portion of the roof of the ventricle. Most of the outflow of CSF from the fourth ventricle passes through this aperture. The cerebral aqueduct is a narrow, curved channel running from the posterior third ventricle into the fourth. Ref: Waxman S.G. (2010). Chapter 11. Ventricles and Coverings of the Brain. In S.G. Waxman (Ed), Clinical Neuroanatomy, 26e. Sample Previous Year Question on Foramen Part III based on previous Year Questions of NEET PG, USMLE,PLAB,FMGE (MCI Screening). Please visit www.medicoapps.org for more such Quizzes A 17-year-old male is examined by a physician, who notes a mass at the back of the young man's tongue. The physician biopsies the mass, and the pathology report comes back with a diagnosis of normal thyroid tissue. The occasional presence of such tissue at the back of the tongue is related to the embryonic origin of the thyroid near which of the following structures? A: First pharyngeal pouch B: Foramen cecum C: Nasolacrimal duct D: Second pharyngeal arch
  • 30. Correct Ans:B Explanation The thyroid gland originates as a mass of endodermal tissue near the foramen cecum, which is near the tuberculum impar (which becomes the central part of the tongue). During development, the thyroid descends in front of the pharynx, maintaining a connection to the tongue via the thyroglossal duct. Usually, the thyroglossal duct disappears. Uncommonly, residual ectopic thyroid tissue can be left anywhere along the path, including at the back of the tongue. (In rare patients, all of the thyroid tissue remains at this site, forming a mass that should not be excised, for obvious reasons!). The first pharyngeal pouch develops into the middle ear and eustachian tube. The nasolacrimal ducts connect the eyes to the mouth. The second pharyngeal arch develops into many muscles of the face and styloid process of the temporal bone. Sample Previous Year Question on Foramen Part III based on previous Year Questions of NEET PG, USMLE,PLAB,FMGE (MCI Screening). Please visit www.medicoapps.org for more such Quizzes A 17-year-old male is examined by a physician, who notes a mass at the back of the young man's tongue. The physician biopsies the mass, and the pathology report comes back with a diagnosis of normal thyroid tissue. The occasional presence of such tissue at the back of the tongue is related to the embryonic origin of the thyroid near which of the following structures? A: First pharyngeal pouch B: Foramen cecum C: Nasolacrimal duct D: Second pharyngeal arch Correct Ans:B Explanation The thyroid gland originates as a mass of endodermal tissue near the foramen cecum, which is near the tuberculum impar (which becomes the central part of the tongue). During development, the thyroid descends in front of the pharynx, maintaining a connection to the tongue via the thyroglossal duct. Usually, the thyroglossal duct disappears. Uncommonly, residual ectopic thyroid tissue can be left anywhere along the path, including at the back of the tongue. (In rare patients, all of the thyroid tissue remains at this site, forming a mass that should not be excised, for obvious reasons!). The first pharyngeal pouch develops into the middle ear and eustachian tube. The
  • 31. nasolacrimal ducts connect the eyes to the mouth. The second pharyngeal arch develops into many muscles of the face and styloid process of the temporal bone. Sample Previous Year Question on Foramen Part III based on previous Year Questions of NEET PG, USMLE,PLAB,FMGE (MCI Screening). Please visit www.medicoapps.org for more such Quizzes Which gland is derived from foramen caecum? A: Pituitary B: Thyroi d C: Thymus D: Parathyroid Correct Ans:B Explanation The site of origin of the thyroglossal duct is the foramen caecum. It grows down in the midline into the neck. Its tip soon bifurcates. Proliferation of the cells of this bifid end gives rise to the two lobes of the thyroid. Ref: Textbook of Human Embryology, 6th Ed Page 119. Sample Previous Year Question on Foramen Part III based on previous Year Questions of NEET PG, USMLE,PLAB,FMGE (MCI Screening). Please visit www.medicoapps.org for more such Quizzes All of the following characteristics differentiate a typical cervical vertebrae from a thoracic vertebrae except A: Mas a triangular vertebral canal B: Has foramen transversarium C: Superior articular facet is directed backwards & upwards. D: Has a large vertebral body Correct Ans:D Explanation
  • 32. The transverse process of cervical vertebrae is pierced by a foramen called the foramen transversarium.The laminae of cervical vertebrae are long(transversely)and narrow (vertically).The spinous processes are short and bifid in a typical cervical vertebra. The vertebral bodies progressively increase in size from above downwards.They are therefore smallest in the cervical vertebrae and largest in the lumbar vertebrae. Ref: Textbook of Anatomy with Colour Atlas by Inderbir Singh By Inderbir Singh Page 85 Sample Previous Year Question on Foramen Part III based on previous Year Questions of NEET PG, USMLE,PLAB,FMGE (MCI Screening). Please visit www.medicoapps.org for more such Quizzes Cervical vertebrae can be differentiated from thoracic vertebrae on the basis of which of the following features? A: Triangular body B: Presence of foramen transversarium C: Superior articular facet which is directed backwards and sidewards D: Presence of a large vertebral body Correct Ans:B Explanation Foramen transversarium is a characteristic feature of cervical vertebrae. Foramen transversarium of transverse process of cervical vertebrae transmits:  Vertebral artery,  Vertebral veins,  Branch from the inferior cervical sympathetic ganglion. Thoracic vertebrae is chracterised by the presence of costal facets in the body. Lumbar vertebrae has:  Kidney shaped body  Long thin transverse process  Spine which is short, quadrilateral and horizontal  Presence of mamillary body and accessary mamillary process  Absence of foramen transvorsorium and costal facets.
  • 33. Sample Previous Year Question on Foramen Part III based on previous Year Questions of NEET PG, USMLE,PLAB,FMGE (MCI Screening). Please visit www.medicoapps.org for more such Quizzes All of the following characteristics differentiate a typical cervical vertebrae from a thoracic vertebrae, EXCEPT: A: Has a triangular vertebral canal B: Has foramen transversarium C: Superior articular facet is directed backwards and upwards D: Has a large vertebral body Correct Ans:D Explanation Cervical vertebrae has a small body. Its transverse diameter is greater than the anteroposterior length. Outline of thoracic vertebral body when viewed from above is heart shaped. Features of a typical cervical vertebrae:  Each transverse process has foramen transversarium.  The spines are short and bifid.  Vertebral foramen is large and triangular. Features of thoracic vertebrae:  Presence of costal facets on each side of the body for articulation with ribs.  Their spinous process are long, slender, and directed downwards.  Vertebral foramina is circular. Ref: TEXTBOOK of ANATOMY and PHYSIOLOGY for NURSES ASHALATA By Jaypee Brothers, page 72. Sample Previous Year Question on Foramen Part III based on previous Year Questions of NEET PG, USMLE,PLAB,FMGE (MCI Screening). Please visit www.medicoapps.org for more such Quizzes Which of the following is the location of the mental foramen?
  • 34. A: Between the premolar of mandible B: First molar of mandible C: Canine of mandible D: Canine of maxilla Correct Ans:A Explanation The mental foramen lies below the interval between the premolar teeth. The mental nerve exits the mandible through the mental foramen and supplies the lower lip and chin. The mental nerve is palpable and sometimes visible through the oral mucosa adjacent to the roots of the premolar tooth. Ref: Gray's Basic Anatomy By Richard Drake, A. Wayne Vogl, Adam W. M. Mitchell, 2012, Page 504. Sample Previous Year Question on Foramen Part III based on previous Year Questions of NEET PG, USMLE,PLAB,FMGE (MCI Screening). Please visit www.medicoapps.org for more such Quizzes